Sunteți pe pagina 1din 58

1. Issue spot/ find the rules its talking about.

2. List options
3. Read rules carefully: look for may v. shall, etc.
4. Apply each rule to the fact pattern (do all of step 3,4, 5 for each rule)
5. Argue both sides of each rule
6. Mention any relevant cases/articles (if have time)
7. Make a determination of outcome, if possible. If its not clear, say so.
How to write answers:
Start essay with a violation of any rule is a violation of Rule 8.4.
1) Issues: (This scenario raises issues under Rule, Rule, Rule, etc)
2) Rule 1.
Could argue Aapply rule to facts
Could argue Bapply rule to facts
Cases/articles
But probably (pick A or B)
3) Rule 2.
Could argue Aapply rule to facts
Could argue Bapply rule to facts
Cases/articles
But probably (pick A or B)
A lawyer must act with dedication to the interests of the client, but is not required to press for every legal
advantage available. The rule specifically caution against acting in a way that is disrespectful or rude to
other members of the legal profession. Here, the lawyer may either [INSERT OPTION 1] or [INSERT
OPTION 2]. Both would be acceptable under 1.3, but [OPTION 1] would [INSERT PRO OR CON].
Conversely, [OPTION 2] would [INSERT PRO OR CON]. Neither option would be technically wrong
under the rules, but [OPTION 1 OR 2] would [INSERT REASON FOR CHOICE]. This option is
probably the favored option under the rules because it avoids using scorched earth tactics, a concern the
drafters specifically tried to avoid when replacing the word zeal with diligence.
Is there an attorney-client relationship? A lawyer has a duty to all clients and prospective clients to
provide
1.1: competence; knowledgeable, skillful, thorough, and effective representation
1.4: communication; informing the client of all facts the client would need to know in order to
make informed decisions about the matter
2.1: candor; a lawyer shall exercise independent professional judgment and render candid
advice
1.6: confidentiality; keeping absolutely secret all information learned in the course of
representing the client, and not using this information in any way that would have an adverse
effect on the client
1.7, 1.8: conflicts; refraining from representing other clients whose interests are opposed to the
first client, and refraining from self-dealing
Formalistic approach: look at whether the language of the rules was actually violated
Purposive approach: ask what harm the rules seek to prevent and balance that against the good that
could be done by permitting certain amount of deception by lawyers
Undercover drug busts are directed by lawyers to ensure the evidence obtained is admissible in court; the
legal system appears to tolerate a great deal of deception in the investigation of criminal wrongdoing and
civil rights violations (i.e. use of testers in housing discrimination cases).

Rule 1.1: Competence (Related rules: 1.2, 1.3, 1.5, 6.2)


Step 1Write: Model Rule 1.1 requires a lawyer to have the legal knowledge, skill, thoroughness, and
preparation reasonably necessary to represent a client. In fact, not only do the model rules demand
competency on the part of the lawyer, but also the 6th Amendment of the Constitution, which guarantees a
right to effective assistance of counsel.
Step 2Does the issue involve a lawyers lack of knowledge about the subject matter of the case?
Write: Rule 1.1 defines competence by breaking it into four elementsknowledge, skill, thoroughness,
and preparation.
A lawyer need not necessarily be experienced in a particular matter in order to be considered competent in
that matter. In this case, although [INSERT LAWYER NAME] was professionally trained and
presumably took and passed the bar, such credentials do not make him/her competent to handle cases in
all areas of the law. In this case, [LAWYER] had [EXPLAIN EXPERIENCE] in this field of law but was
arguably not competent in this area of the law because [INSERT REASON]. According to comment 2,
a lawyer need not necessarily have special training or prior experience to handle legal problems of a type
with which the lawyer is unfamiliar [if the] lawyer can provide adequate representation in a wholly
novel field through necessary study. According to the rules, a lawyer is not required to know the law
that governs the clients legal claim before undertaking representation s long as the lawyer is able to
acquire the necessary knowledge with reasonable diligence. Thus, [LAWYER] could have cured the
situation by doing the necessary research and consulting with others to learn about this area of law.

Knowledge of legal principles: A lawyer is expected to be familiar with the principles of the law
applicable to a clients needs.
o Basic Research: A lawyer must be able to find applicable rules of law, whether or not commonly
known or settled using standard research sources.
o Knowledge of procedure: A lawyer is required to know and follow all relevant rules of procedure
applicable to matters undertaken.
Skill: A lawyer is expected to have basic skills including logical thinking, drafting, and arguing.
o Drafting: Skills required of a lawyer include the ability to draft pleadings & documents.
o Legal Analysis: Competence includes the ability to analyze relevant rules and principles and
apply them to clients circumstances.
Thoroughness and Preparation
o Investigation and Research: The interrelated obligations of thoroughness & preparation require a
lawyer to investigate all relevant facts & research applicable law.
o Application to client matters: After learning the relevant law and facts, a lawyer must then be able
to apply them to the clients matters.
Step 3Is the lawyer inexperienced?
New lawyers: Lack of competence may be a particular problem for a new or inexperienced lawyer.
See Rule 1.5 (Responsibilities of Partners, Managers, and Supervisory Lawyers)
Unfamiliarity with areas of law: Competence includes the ability to discern when an undertaking
requires specialized knowledge or experience that a lawyer does not have and requires that the lawyer
either acquire the expertise, associate with a specialist, or decline the undertaking and refer it to a
competent specialist.
Self-education in unfamiliar area: Comment 2 notes, it is possible for a lawyer to provide competent
representation in a wholly novel field through necessary study, to the extent the lawyer must spend
excessive amounts of time preparing for tasks that with experience become routine, the lawyer should
not expect the client to pay for excessive time the lawyer spends educating himself. See Rule 1.5.
o There are a number of factors the rules recommend when determining whether an inexperienced

attorney in a particular field may competently represent a client in that field. Consider whether
the lawyer has any transferable skills that she can carry over from areas in which she is
competent, including:
The relative complexity and specialized nature of the matter
The lawyers general experience
The lawyers training and experience in the field in question
The preparation and study the lawyer is able to give the matter
Ability to consult with a lawyer of established competence in the field in question
Sixth Amendment considerations: Incompetent representation by criminal defense counsel can violate
the Sixth Amendment right to effective assistance of counsel.
o See Washington v. Strickland, 2-prong test; pp. 304.
Public defenders and appointed counsel: The obligation to be adequately prepared may justify a
refusal to proceed with the defense of a criminal case is a court-appointed lawyer or public defender
has not had an adequate opportunity to prepare. (Note: a potential violation of Rule 1.1 may be good
cause for a lawyer to seek to avoid appointment by the court.) See Rule 6.2
Heavy Caseloads: The duty to provide competent representation requires a lawyer to keep his
caseload manageable to allow adequate time and effort for each client matter. When heavy caseloads
doe to circumstances beyond the lawyers control jeopardize their ability to render competent
representation, the lawyer may be obligated to ask to withdraw. If the court denies a motion to
withdraw however, a lawyer must continue the representation to the best of his ability. See Rule
1.16(c).
A client must be properly informed from the outset whether or not his lawyer will need to
refer the matter to another attorney who is established in the related field.
See William Manger case, pp. 281
Step 4Was there an emergency?
Because competence is a function of reasonableness, a different standard of competence applies in
an emergency. A lawyer in such cases may give advice reasonably necessary in the circumstances
where referral to or consultation with another lawyer would be impractical. However, the lawyer
should limit the legal services provided to those necessary in light of the emergency circumstances.
Essentially, the emergency exception to the duty of competence requires that the lawyer enter a
limited engagement with the client to protect the interests of the client caused by the emergency.

Rule 1.2: Scope of Representation (Related rules: 1.1, 1.4, 1.8, 1.14, 1.16, and 5.6)
Although 1.2 defines the scope of representation; there are limitations to actions a lawyer can take at the
direction of his client. For example, in Jones v. Barnes (pp. 338) a client directed his attorney to file a
number of claims the lawyer deemed frivolous. The court applauded the lawyers decision not to file
superfluous claims noting that it is a lawyers professional duty to argue issues according to the lawyers
judgment as to their merit while also supporting his clients appeal to the best of his ability.
(a) [1] Subject to paragraphs (c) and (d),
[2] a lawyer shall abide by a client's
decisions [3] concerning the objectives
of representation and, [4] as required by
Rule 1.4, [5] shall consult with the
client as to the means by which they are
to be pursued. [6] A lawyer may take
such action on behalf of the client as is
impliedly authorized to carry out the
representation. [7] A lawyer shall abide
by a client's decision whether to settle a
matter. [8] In a criminal case, the
lawyer shall abide by the client's
decision, after consultation with the
lawyer, [9] as to a plea to be entered,
whether to waive jury trial and whether
the client will testify.
(b) [1] A lawyer's representation of a
client, including representation by
appointment, [2] does not constitute an
endorsement of the client's political,
economic, social or moral views or
activities.
(c) [1] A lawyer may limit the scope of
the representation [2] if the limitation is
reasonable under the circumstances and
[3] the client gives informed consent.
(d) [1] A lawyer shall not counsel a
client to [2] engage, or assist a client, in
conduct that [3] the lawyer knows is
criminal or fraudulent, [4] but a lawyer
may discuss the legal consequences of
any proposed course of conduct with a
client and [5] may counsel or assist a
client to make a good faith effort to
determine the validity, scope, meaning
or application of the law.

Lawyer must defer to client about objectives and must


consult about means. (Means: decisions that are
procedural or tactical in nature)
Lawyer must pursue specific objectives for which the
lawyer was retained. (Objectives: decisions that directly
affect the ultimate resolution of the case or substantive
rights)
Comment 2: disagreements about representation
Dictates that the client has the right to define the objectives
of representation and gives the client the explicit right to:
To settle or refuse to settle (in a civil case),
To decide on the plea to be entered (in criminal cases),
Whether to waive a jury trial, and
Whether to testify
Note: Comment 1 allows the lawyer to defer to the client
regarding questions concerning third parties who might be
adversely affected.
Makes clear that a lawyer who represents a client does not
necessarily agree or endorse that clients views. This point
was exemplified in Abraham Sofaers case when he
represented Moammar Gadhafi, a client accused of
terrorism, although he certainly did not hold Gadhafis
anti-American views.
Comment 6: Allows a lawyer to limit the scope of
representation if the client gives informed consent.
Comment 7: Limitation must be reasonable.

Inquiring into clients affairs: assistance in unlawful


conduct not excused by a failure to inquire into clients
objectives.
Permits a lawyer to assist a client in making a good-faith
determination of the validity, scope, meaning, or
application of the law.
o A lawyer representing a client in preparation of a case
may advise the client of the most favorable positions if
the lawyer has a good-faith argument for an extension,
modification, or reversal of existing law
o That good-faith belief can exist even if the lawyer
believes the position will not prevail
o That good faith requires some realistic possibility of
success.

For issues not explicitly stated in the rules, a lawyer may take actions that are impliedly authorized.
Although the rules do not define what constitutes a lawyers implied authority, [INSERT ISSUE] is
arguably within such authority. As noted in the comments to 1.2, clients normally defer to the special
knowledge and skill of their lawyer since [INSERT ISSUE] is they kind of special knowledge that is
know by a lawyer and not by a lay client, it would be reasonable for the lawyer here to defer to his own
judgment in deciding [INSERT ISSUE]. This conclusion is further cemented by Restatement 21,
comment e, which states that a lawyer may take any lawful measure within the scope of representation
that is reasonably calculated to advance a clients objectives. It seems clear that the lawyers decision to
[INSERT ISSUE] is within the purview of both Rule 1.2 and the Restatement.
Lawyer must abide by clients decision regarding settlement of claims: a lawyer has no inherent
power to settle a clients claim. Accordingly, this rule requires a lawyer to get the clients specific
authorization to enter a settlement agreement on the clients behalf. When there are multiple
clients represented in a single action, Rule 1.8(g) requires that each client consent to any
aggregate settlement.
Lawyer may not coerce client to approve settlement
o Fee agreement may not be used to deprive client of right to approve settlement: although
a client may grant the lawyer express authority to settle, a retainer agreement that forbids
the client from settling a case without the lawyers consent or that creates a financial
disincentive to do so may be against public policy and impermissible under 1.2(a).
o Withdrawal from case if client fails to settle: similarly, a lawyer who withdraws from a
case due to a clients unwillingness to settle may forfeit his entitlement to a fee.
Representing clients with diminished capacity: when a clients ability to make decisions about the
representation is diminished, the division of decision-making authority between the lawyer and
client may be different. See Rule 1.14.

Rule 1.3: Diligence (Related rules: 1.2, 1.4, 1.16)


Diligence refers to the timeliness aspect of competence. Lawyers are obligated to be diligent on their
clients behalf; as Rule 1.3 states, a lawyer shall act with reasonable diligence and promptness in
representing a client. Unreasonable delay by a lawyer may give rise to diligence issues.
Step 1: Has the lawyer taken whatever lawful and ethical measures are required to vindicate a clients
cause or endeavor?
A lawyer must act with dedication to the interests of the client, but is not required to press for
every legal advantage available. The rule specifically caution against acting in a way that is
disrespectful or rude to other members of the legal profession. Here, the lawyer may either
[INSERT OPTION 1] or [INSERT OPTION 2]. Both would be acceptable under 1.3, but
[OPTION 1] would [INSERT PRO OR CON]. Conversely, [OPTION 2] would [INSERT PRO
OR CON]. Neither option would be technically wrong under the rules, but [OPTION 1 OR 2]
would [INSERT REASON FOR CHOICE]. This option is probably the favored option under the
rules because it avoids using scorched earth tactics, a concern the drafters specifically tried to
avoid when replacing the word zeal with diligence.
Step 2: Is the lawyers workload too burdensome?
The rules require a lawyer to control his workload so that each matter can be handled
competently. A lawyer who has a full docket for example, should not take on a new client. This
part of the rule is often implicated when a lawyer has a substance abuse problem or when a
lucrative client approaches the lawyer about representation. In the latter situation, a lawyer must
inform the potential client about his limitations and potentially refuse the work.
Step 3: Has the lawyer procrastinated on his duties?
The rules specifically limit procrastination and cite missed deadlines and sloppy work as potential
consequences of such procrastination. Even without missing a formal deadline, a lawyer may be
subject to discipline under 1.3 for unreasonable delay.
Failure to take any action: It is the duty of diligence that requires a lawyer to actually perform the
work for which he was retained. A lawyer who take no action on a case is subject to discipline
under 1.3
Failure to follow through or complete client matters: 1.3 imposes a duty to carry through to
conclusion all matters related to the representation, unless the lawyer withdraws.
Inadequate investigation and preparation: requires an adequate level of preparation and
investigation for each particular representation.
Step 5: Is the lawyer-client relationship over?
If a client moves firms or otherwise wished to end a relationship he has a duty give notice of
withdrawal. Even when a lawyer has resolved a matter, a prudent lawyer will give notice so that
the client does not rely on the lawyers representation in subsequent appeals or other matters. The
rule suggests this be done in writing.
Step 6: Has the lawyer died or otherwise incapacitated?
The rule advises a solo practitioner to prepare a plan, in conformity with the rules, that designates
another competent attorney to handle client files upon death or disability.

Rule 1.4: Communications (Related rules: 1.2, 1.3, 1.7, 1.13, 1.14, 3.4, 7.1)
Step 1Write: The Model Rules require lawyers to communicate with clients. This is because the
communication duty is critical to maintaining a quality lawyer-client relationship & to keep clients
reasonably informed as to the status of their case.
Step 2: Did the lawyer promptly inform and reasonably explain to the client of any decision or
circumstance where informed consent from the client is required?
Rule 1.3 requires informed consent about issues which a client must make a decision, about the
status of a matter, and about matters which the client has requested information. (See pp. 427)
Step 3: Did the lawyer reasonably consult with the client about the means with which the clients
objectives are to be accomplished? (See also 1.2 regarding the scope of the relationship.)
The client should have sufficient information to participate intelligently in decisions concerning
objectives of representation. Rule 1.3 requires a lawyer to discuss with the client the means by
which the clients objectives (as defined by 1.2) are to be met. 1.4(b) also requires a lawyer
explain the law and the benefits and risks of alternative courses of action
Step 4: Did the lawyer keep the client reasonably informed about the status of the matter?
Trial developments: lawyer should inform his client of developments. A lawyer is not required to
inform the client of every minute detail, but major trial developments should be reported to the client.
Comment 3, a lawyer must communicate significant developments affecting the timing or substance
of the representation.
Step 5: Did the lawyer promptly comply with reasonable requests for information?
Lawyer must communicate promptly: Ideally, a lawyer will keep a client informed such that the client
will not have to affirmatively request information. The rules do not clarify whether a lawyer who is
not currently representing a client must promptly respond to requests for information, however, as a
matter of good business practices, a lawyer should probably respond to even non-urgent matters of
regular clients, or, should acknowledge receipt of the question & tell the client when the client should
expect to receive an answer.
Attempts to cover up: if a lawyer errs or fails to follow client instructions on a material matter, the
rule requires the lawyer to inform the client
Settlement offers: 1.2 and 1.4 require a lawyer to communicate a settlement offer unless the lawyer is
aware from prior discussions that the client would reject the offer.
Criminal Representation: duty to consult with client about issues over which a client has decisionmaking authority is imposed by 1.2 but it also a function of defendants constitutional rights.
Missing clients: even when a lawyer loses track of a client the duty to communicate important
information persists & requires at least reasonable effort to find client to deliver the information
Communicating through others: a lawyer may not delegate to subordinates the obligation to
communicate with a client
When a lawyer leaves a firm or the practice of law: must notify their clients
Step 6: Did the lawyer consult with the client about any relevant limitation of the lawyers conduct?
See 1.3 (diligence and workload)
Withholding information from a client is rarely ethical, however, Rule 1.4 does not explicitly require
lawyers to be honest with their clients. Model Rule 1.4, Comment [3] describes three scenarios in which
withholding information would be proper:
(1) When the lawyer reasonably believes that immediate communication of information to client
would harm the client. In these cases, the lawyer may delay communicating the information.
o I.e.: a psychiatrist tells you not to discuss a diagnosis w/ client, b/c disclosure could be
harmful. You should heed psychiatrists warning & not disclose the diagnosis.
(2) When the client would be likely to react imprudently to immediate communication
(3) When a court rule or order governing litigation requires that the lawyer not disclose the

information to the client.

Rule 1.5: Fees (Related rules: 1.2, 1.4, 1.8)


Step 1: Is the fee reasonable?
Under Model Rule 1.5, the fee regulations require that a lawyers fee not be clearly excessive.
Factors considered in determining whether a fee is unreasonable, though not exhaustive, are:
o The time and labor required, the novelty and difficulty of the questions involved, and the skill
requisite to perform the legal service properly; (1.5(a)(1))
o The likelihood, if apparent to the client, that the acceptance of a particular employment will
preclude other employment by the lawyer; (1.5(a)(2))
o The fee customarily charged in the locality for similar legal services; (1.5(a)(3))
o The amount involved and the results obtained; (1.5(a)(4))
o The time limitations imposed by the client or by the circumstances; (1.5(a)(5))
o The nature and length of the professional relationship with the client; (1.5(a)(6))
o The experience, reputation, and ability of the lawyer(s) performing the services; (1.5(a)(7))
o Whether the fee is fixed or contingent; (1.5(a)(8))
Step 2: Is the fee contingent on the outcome of the representation in a criminal or divorce matter?
With a few exceptions, a lawyer is permitted to charge a fee that is contingent on the outcome of the
matter, so long as the contingency fee is not unreasonable.
A lawyer is prohibited, however, for charging a contingency fee to a defendant in a criminal matter.
A lawyer is prohibited from charging a contingent fee in a domestic relations case when the fee is
contingent on obtaining a divorce or on the amount of alimony recovered.
1.5 (c) requires contingency fees must be in writing
Step 3: Is the fee agreement in a writing signed by the client?
Although ordinarily a fee agreement need not be in writing, a contingent fee agreement must be
in writing. The written agreement must explain the way in which the fee will be calculated an the
way in which deductions for expenses will be calculated.
Step 4: Is the fee shared with nonlawyers?
Fees may not be shared with nonlawyers except under very limited circumstances.
Lawyers within the same firm routinely share fees with one another. However, when lawyers
who are not members of a firm share fees, or when lawyers seek to share fees with nonlawyers,
potential ethical problems arise.
1.5 requires that the client agree in writing to the participation of each lawyer, including the share
each will receive
Step 5: Did the lawyers split the fee proportionally or both undertake full responsibility for the
representation?
The fee-splitting lawyers must share the fee either in proportion to the services rendered by each
lawyer or in some other proportion if the lawyers agree with the client in writing that the lawyers
shall have joint responsibility for the representation.
Step 6: Did the lawyer do any of the following?
Charge fees for doing nothing
Doing remedial work
Charge fees for doing very little
Too many lawyers working on matter
Doing very little and doing it badly
Charging lawyer rates for nonlawyer
work
Doing way too much
Step 7: Is there a dispute over the fee arrangement?
Comment 9 urges lawyers to voluntarily submit to arbitration or mediation to resolve fee disputes

Rule 1.6: Client Confidences (related rules: 1.8, 1.9 (former clients), 1.16, 1.18 (prospective
clients), 3.4)
Rules that dont override 1.6:
Rules that do override 1.6:
1.2(d): counseling fraud
3.3(c): candor to tribunal
4.1(b): failure to disclose to avoid assisting
1.13 (c): whistleblowing in company
in fraud
8.1: correcting mistake in bar application
8.3: reporting professional misconduct
Write: compliance with 1.6 requires not only that lawyers avoid improperly disclosing protected
information, but also that they act competently to preserve confidentiality. Protecting such
confidential information between the attorney and client is meant to encourage a client who needs
legal advice to tell the lawyer the truth; without full information, a lawyer is less able to assist the
client.
To determine if relationship is protected, courts look to intent behind a communication. Two
individuals who never met may enjoy a protected relationship if a person confides in a stranger
attorney for purposes of obtaining legal advice; even if they had no prior contact, they still might
enjoy a protected relationship.
In certain circumstances, lawyers may (but are not required to) reveal confidential information to
prevent future crimes or harms by clients. The model rules focus on preventing future crimes and
preventable harms, which may be the result of clients past acts. See Robert Garrow case on pp.
174.
If the crime in question is over, and the lawyer cannot prevent the harm by revealing it, then no
affirmative duty to reveal information exists. Our belief in the fair administration of justice and
the right to legal counsel tends to lend itself to the conclusion that lawyers should protect as
confidential, most information about past criminal activities by clients.
Relationship to attorney-client privilege

(a) [1] A lawyer shall not reveal information [2]


relating to the representation of a client [3]
unless the client gives informed consent, the
disclosure is impliedly authorized in order to
carry out the representation or the disclosure is
permitted by paragraph (b).

The ethical duty is extremely broad: it protects


from disclosure all information relating to the
representation, and applies at all times.
The attorney-client privilege is more limited; it
protects substance of a lawyer-client
communication from compelled disclosure
made for the purpose of obtaining or imparting
legal advice or assistance, & applies only in
legal proceedings governed by FRE.
Previously disclosed or publicly available
information: Contains no exception permitting
disclosure of information previously disclosed
or publically available. But see 1.9(c).
Disclosure of clients identity: Prohibits
disclosure of a clients identity unless the client
consents or disclosure is impliedly authorized.
Disclosures within firm: Impliedly authorized
to discuss with other firm lawyers unless

(b) [1] A lawyer may reveal information relating to


[2] the representation of a client [3] to the
extent the lawyer reasonably believes [4]
necessary:
1) to prevent reasonably certain death or
substantial bodily harm;
2) to prevent the client from [5] committing a
crime or fraud [6] that is reasonably certain
to result in [7] substantial injury to the
financial interests or property of another
and [8] in furtherance of which the client
has used or is using the lawyer's services;
3) to prevent, mitigate or rectify [5]
substantial injury [6] to the financial
interests or property [7] of another that is
[8] reasonably certain to result or has
resulted [9] from the client's commission of
a crime or fraud [10] in furtherance of
which [11] the client has used the lawyer's
services;
4) to secure legal advice [5] about the
lawyer's compliance with these Rules;
5) to establish a claim or defense on behalf of
the lawyer in a controversy between the
lawyer and the client, [5] to establish a
defense to a criminal charge or civil claim
against the lawyer [6] based upon conduct
[7] in which the client was involved, or to
respond to allegations [8] in any
proceeding concerning the lawyer's
representation of the client; or
6) [5] to comply with other law or a court
order.
Comments
C3: rule applies to any information related to
the representation, regardless of the source.
C4: lawyer cant even provide information that
could reasonably lead to the discovery of
protected information

prohibited by c.5.
Client with diminished capacity: See 1.14(c).
Informed consent: required when disclosure of
information is not impliedly authorized.
Unlike conflicts waivers, informed consent
need not be in writing.
Prospective clients: extends to prospective
clients who consults a lawyer in good faith for
the purpose of obtaining legal advice
Former clients: extends after relationship ends
Organization as a client: See 1.13
Prevent bodily harm: authorizes disclosure to
prevent accidental, but serious, physical harm
that is reasonably certain to occur (i.e. toxic
torts, and the extremely violent nature of prison
life, drug companybirth defects, cancer;
monetarysuicide (Enron Exec Clifford
Baxter), inability to retire).
Economic crimes and frauds: See (b)(2).
Past crimes: absent authority to disclose, a
lawyer may not disclose a clients wrongful
past actions to prevent their continuing harms.
Prevent, mitigate, or rectify injury: See c.8
Crime-fraud exception: a lawyer is not barred
from disclosing otherwise privileged
information about a client who consults the
lawyer to further the commission of a crime or
fraud. See 1.16, & c.7.
Disclosure required by 3.3: a lawyer may be
required by 3.3 to reveal to the court
information otherwise protected to avoid
assisting a client in perpetrating a crime or
fraud. See cc.10-11.
Securing legal ethics advice: although
disclosure to secure ethics advice may not be
impliedly authorized, 1.6(b)(4) permits
disclosure even without implicit authorization
because of the importance of a lawyers
compliance with the model rules.
Disclosure to support claim or defense against
client: See 1.6(b)(56); see also c.14.

C14: disclosing information adverse to client


should be no greater than the lawyer believes is
reasonably necessary.
C17: must take reasonable precautions to
prevent info from going to unintended

recipients. Reasonableness: 1) sensitivity of


information, 2) a confidentiality agreement.
MR 4.4 - Inadvertent Disclosure the client may lose the privilege of confidentiality by
inadvertently disclosing otherwise privileged communications to non-privileged recipients.
Moreover, if a client voluntarily reveals a portion of his privileged communications, courts typically
find that he may not withhold the remainder. If a document relating to the representation of the
lawyers client was inadvertently sent to the lawyer, he must promptly notify the sender. MR 4.4(b).
Rule 1.7 Conflicts of Interest
Model Rule 1.7, the general rule for conflicts of interest, holds that a lawyer must not represent a
client if the representation of that client will be directly adverse to the interests of another client
unless:i) the lawyer reasonably believes that his representation will not adversely affect the
relationship with the other client; andii) each client consents after consultation.
Is there an interest that threatens a lawyers independent professional judgment?
Lawyers must refrain from establishing relationships where a potential conflict of interest will
detrimentally affect the quality of the representation. Conflicts of interest may have negative effects
on a lawyers ability to exercise independent and professional judgment.
Is the lawyer doing business with the client?
Consequences may ensue when a lawyer tries to acquire property rights that compromise his interest
in protecting his clients rights. In conducting other business enterprises with a client, a lawyer
should consider the ramifications of a lawyer entering into a business transaction with a client. A
dispute regarding that business interest could leave client and lawyer in a dangerous adversarial
position, whereby the lawyers property interests could cloud his judgment in his representation of the
client.
Far removed, but potential conflict?
Given that the family members are not going to be directly associated with the case, it probably will
mean nothing to the clients. Some observers might reason that the lawyers need not comply with the
formal requirements of Model Rule 1.7 (getting a formal consent from clients after consultation)
because the relationship between the lawyers involved is so remote in the context of the case at bar.
Nonetheless, an ethical and scrupulous attorney would probably let the clients know that there is a
remote conflict of interest. Chances are a client will not mind but its best to let the clients know
about a potential conflict of interest prior to the start of the attorney-client relationship, rather than
after the proceeding has begun.
Rule 1.7(b) requires both reasonable belief that the lawyer will be able to provide competent and
diligent representation and informed consent, confirmed in writing. Here
As to the consent element, each affected client must consent. This includes former clients as well
as current ones but not third parties. The consents must be "informed", a term defined in RPC
1.0(e):
o "'Informed consent' denotes the agreement by a person to a proposed course of conduct after
the lawyer has communicated adequate information and explanation about the material risks
of and reasonably available alternatives to the proposed course of conduct."
The consents must be "confirmed in writing, after full disclosure and consultation" and if the
conflict involves multiple clients in a single matter the consultation "shall include an explanation
of the common representation and the advantages and risks involved". The rule does not specify
what is to be included in each client's written confirmation. Presumably, as a minimum, the
writing need only contain a simple statement of the facts constituting the conflict, a reference to
there having been a consultation with the lawyer, and a confirmation of consent by the client.

The client should also sign the written document. The rule does not state when the consent must
be obtained, but again, presumably, the consent should be obtained before the conflicting
representation commences.
Turning now to the second curing element, the lawyer must hold the belief that he or she will be
able to provide "competent and diligent" representation to each affected client. "Competence"
and "diligence" are duties imposed by RPCs 1.1 and 1.3.

1.7(a)(1): Directly adverse interests


Representing opposing parties in same lawsuit: is prohibited by 1.7(a)(1) and is not waivable.
Representing someone in unrelated suit against existing client: 1.7(a)(1) prohibits a lawyer from
representing anyone directly adverse to a current client, even if the matters are unrelated.
Disqualification not inevitable: a violation of 1.7(a) does not always result in disqualification,
particularly if the firm has implemented a screen and the complaining party cannot show it was
harmed.
Hot-potato rule: a law firm trying to take on a matter that presents a conflict under 1.7 may not
simply drop a client to be free to take on a more attractive one. Courts are more forgiving if the
conflict was unforeseeable and arose through no fault of the law firm (i.e. when a conflict is
created by a merger or acquisition).
Simultaneously representing clients involved in different suits over related matters: may give rise
to a conflict.
Direct adversity in nonlitigation context: comment 7 points out that direct adversity may occur
outside the litigation context. In cases that involve transactional matters, a material limitation
under 1.7(a)(2) is more common.
1.7(a)(2) (Material limitation conflicts): focuses not on direct adversity of interests, but on the extent
to which a representation is likely to be limited because of interests jeopardizing the lawyers exercise
of independent professional judgment.
Responsibilities to other clients: multiple representation that does not involve direct adversity of
interests under 1.7(a)(1) can still pose a conflict under 1.7(a)(2) if responsibilities to one client
could materially limit the representation of another.
Duties to former clients and prospective clients: 1.9 and 1.18 addresses conflicts arising out of
duties to former and prospective clients.
Positional conflicts: arise when a lawyers successful advocacy of a clients legal position in one
case could be detrimental to the interest of a different client in another case.
Corporate-family conflicts: majority rule is that there is no per se prohibition against undertaking
a representation that is adverse to an affiliate of a corporate client. In some cases, however, the
relationship between the parent and subsidiary is too close to permit the firm to proceed. See
1.13(g)
Lawyers own financial and professional interests: both financial and professional interests may
materially limit representation. When a lawyer for a corporation serves on the corporations
board of directors, the duties may come into conflict. See 1.8(a) and 1.5
Lawyers family ties and personal relationships: may pose material limitations
Sex with clients: a relationship with a client that arises during the course of the representation can
interfere with the lawyers ability to exercise independent professional judgment on the clients
behalf. This is a specific instance of a material limitation conflict.
Responsibilities to others: responsibilities attendant upon other kinds of relationships, in addition
to personal and lawyer-client relationships can create material limitation conflicts.
1.7(b) (informed consent): if a lawyer reasonably believes that no client will be adversely affected,
and if the representation is not prohibited by law and does not involve one client asserting a claim

against another, the lawyer may represent conflicting interests if each affected client gives informed
consent, confirmed in writing. Rule 1.0(e) defines informed consent to mean that the lawyer has
communicated adequate information and explanation about the material risks of and reasonably
available alternatives to the proposed course of conduct.
Written confirmation: 1.7(b)(4) requires a lawyer to obtain written confirmation of a clients
informed consent to the lawyers conflict of interest. See comment 20.
Prospective waivers: the effectiveness of a clients prospective waiver of a conflict depends upon
whether the conflict is consentable in the first place, and how clearly the waiver identifies the
anticipated conflict.

Rule 1.8: Conflict of Interest: Current Clients: Specific Rules

1.8(a) (business transactions between client and lawyer): prohibits business transactions between a
lawyer and a client unless the lawyer complies with specific conditions designed to protect the client.
1.8(a) does not apply to ordinary client-lawyer fee arrangements, which are governed by 1.5, but does
apply to lawyers who sell their clients goods or services related to the practice of law, such as title
insurance or investment services; its requirements must be met even in the transaction is not closely
related to the subject matter of the representation. If 1.8(a) applies
1.8(a)(1): the transaction must be objectively fair and reasonable to the client
1.8(a)(1): contract terms are fully disclosed and transmitted in a manner that can be reasonably
understood by the client
1.8(a)(2): the client must be advised in writing of the desirability of seeking the advice of
independent counsel and
1.8(a)(2): client must be given a reasonable opportunity to do so
1.8(a)(3): client gives informed consent, in writing, to the essential terms of the transaction and
the lawyers role in the transaction, including whether the lawyer is representing the client in the
transaction
Loans: and other situations where sales and investment transactions may unfairly favor the lawyer
between lawyers and clients are governed by 1.8(a)
Ownership and investment as fee payment: according to comment 1, 1.8(a) applies when a lawyer accepts
an interest in a clients business or other nonmonetary property as payment of all or part of a fee. See
also, comment 4 and rule 1.5
Changing fee agreements: 1.8(a) does not apply to ordinary client-lawyer fee agreements, it has been
applies to efforts to modify agreements during the course of the representation.
Referral agreements: 1.8(a) applies to a lawyers receipt of compensation for referring a client to a
nonlawyers professional such as an investment advisor; some jurisdictions believe compensated referrals
are improper even with full disclosure and consent

1.8(b) (use of representation related to representation): governs the use of information relating to the
representation of a current client. Once a client-lawyer relationship has ended, 1.9 governs both
disclosure and use of the information. 1.8(b) prohibits a lawyer from using information relating to the
representation of a client to the clients disadvantage without the clients informed consent unless
permitted or required by other rules. 1.8(b) however, does not prohibit a lawyer from using client
information in a way that does not disadvantage a client.
1.8(c) (gifts to lawyers): prohibits a lawyer from either soliciting a substantial gift from a client or
preparing an instrument by which a client gives a substantial gift to the lawyer or a relative of the
lawyer, unless the client and recipient are themselves relatives (or very close friends). See also 1.7.
1.8(d) (literary rights): a lawyer who is representing a client is prohibited from making or negotiating
an agreement giving the lawyer literary or media rights to an account based in substantial part upon
information relating to the clients representation. See comment 9.
1.8(e) (financial assistance to clients): prohibits a lawyer from providing financial assistance to a
client in connection with pending or contemplated litigation. The lawyer may neither lend money nor
guarantee a third-party loan to a client. Except:
1.8(e)(1): permits a lawyer to advance court costs and litigation expenses and to condition
repayment upon recovery
1.8(e)(2): permits a lawyer for an indigent client to pay court costs and litigation expenses
outright
Most jurisdictions do not find any implicit humanitarian exception for helping needy clients, but a

lawyer may argue that expenses of litigation include medical expenses necessary for a client to
have his day in court or a nominal amount is not a gift in connection with representation. See
comment 10.
1.8(f) (person paying for lawyers services): imposes three conditions on acceptance of third-party
payment:
the client must give informed consent;
there must be no interference with the lawyers independence of professional judgment or with
the client-lawyer relationship; and
information relating to the representation must be protected as required by 1.6
Note: 5.4(c) prohibits a lawyer from permitting the person paying for the lawyers services to direct
or regulate the lawyers professional judgment. See comments 11 & 12, 1.6, 5.4(c) & 1.7.
1.8(g) (aggregate settlements or plea agreements): if a lawyer is participating in an aggregate
settlement of claims by or against multiple clients or, in a criminal aces, an aggregated agreement for
guilty or nolo contedere pleas. 1.8(g) required the lawyer to obtain informed consent of each client,
confirmed in writing and signed by each client, after the lawyer has disclosed the existence and
nature of all the claims or pleas involved and of the participation of each person in the settlement.
1.8(h) (limiting liability and settling malpractice claims):
1.8(h)(1) prohibits agreements prospectively limiting a lawyers liability to clients for malpractice
unless the client is independently represented in making the agreement. Comment 14 provides
that 1.8(h):
o does not prohibit agreements to arbitrate legal malpractice claims
o does not prohibit lawyers from practicing in limited-liability entities
o does not prohibit agreements limiting the scope of the representation in accordance with rule
1.2.
1.8(h)(2) prohibits a lawyer from settling a claim or potential claim with an unrepresented client
or former client unless that person is advised in writing of the desirability of seeking independent
counsel, and is given a reasonable opportunity to do so.
1.8(i) (acquiring proprietary interest in litigation): restricts a lawyers ability to acquire a proprietary
interest in a clients cause of action or the subject matter of litigation. See comment 16.
1.8(j) (client-lawyer sexual relationships): prohibits all client-lawyer sexual relationships, including
consensual relationships, except those predating the formation of the client-lawyer relationship. See
also comment 18, 19, and 1.7(a)(2)
1.8(k) (imputation of prohibitions): imputes all of 1.8s prohibitions to every associate firm lawyer.

Rule 1.9: Duties to Former Clients


Write: 1.9 prohibits opposing a former client only if the matter is "substantially related" or if the
lawyer uses information relating to the representation to the former client's disadvantage.

1.9(a) (when proposed representation would be disloyal to former client): provides that unless the
former client gives informed consent, confirmed in writing, the lawyer may not represent anyone with
materially adverse interests in the same or substantially related matter. Courts evaluating whether a
lawyer is permitted to undertake the subsequent representation consider four criteria:
Whether there was a lawyer-client relationship in the first place; that is whether the lawyers
actually represented the client and, if so, whether the relationship is over;
Whether the subsequent representation involves the same or substantially related matter;
If the matters are either the same or substantially related, whether the interests of the subsequent
client are materially adverse to those of the former client; and
Whether the former client consented or, in the disqualification context, waived objection to the
new representation.
Was there a lawyer-client relationship? Need not be explicit but may be implied from circumstances
Organizational clients: when the lawyer has represented an organization, rather then an
individual, it is not always clear who enjoys former-client status for purposes of 1.9. Three areas
typically cause confusion:
o Corporate affiliates: because the primary concern about representation adverse to a
former client is the risk that confidential information of the former client will be used to
its disadvantage, courts are likely to disqualify a lawyer who had access to confidential
information of the corporate affiliate during the representation.
o Organizations and their individual constituents: an implied lawyer-client relationship with
an individual officer or employee may arise as a result of the lawyers course of dealings
with that individual during the representation of the organization.
o Mergers and transfers of assets
Insurance defense representation
Prospective clients: when a lawyer-client relationship is formed, the lawyer is disqualified from
representation adverse to the former client in a same or substantially related matter. The rule does
not require a showing that the lawyer received significant confidential information. When a
person consults with a lawyer but never forms a lawyer-client relationship, the person is a
prospective client whose confidential information is entitled to protection, but the lawyer is
disqualified from subsequent adverse representation only if the lawyer received information
from the prospective client that could be significantly harmful to that person in the matter. See
also 1.18.
Is the client a former client?
1. Once a lawyer-client relationship has been found to exist, the next issue is to determine whether the
relationship has ended or is ongoing. Rule 1.9 applies only to former clients; conflicts of interest
involving current clients are governed by rules 1.7 and 1.8, which are more restrictive than rule 1.9.
When a lawyer is retained for a specific matter, the representation terminates when the matter has
been resolved. See 1.3, comment 4.
Are the matters substantially related?
2. Comment 1 makes clear that a lawyer could not properly seek to rescind on behalf of a new client a
contract drafted on behalf of the former client and a lawyer who has prosecuted an accused person
could not properly represent the accused in a subsequent civil action against the government
concerning the same transaction. Similarly, a lawyer who has previously represented multiple

clients in a matter may not represent one of them against another in the same or substantially related
matter after a dispute arises among them.
3. Same transaction or legal dispute: according to comment 3, two matters are by definition substantially
related if they involve the same transaction or legal dispute.
4. Incidental similarities: between the transactions or disputes do not create a substantial relationship.
5. Different transaction or legal dispute: matters that involve difference transactions or disputes will be
deemed substantially related if there is substantial risk that protected information as would
normally have been obtained in the first representation would materially advance the new clients
interests. Focusing on the riskas opposed to the actual factof disclosure spares the former client
who is seeking disqualification from having to disclose the very information it is trying to protect.
Courts consider both the factual and the legal similarities between the two matters.
1. As comment 3 notes, the general knowledge of an organizational clients policies and
practiceswhat their typical business strategies are, whether they are aggressive or
conservative in legal disputes, etc.does not in itself require disqualification.
Are clients interests materially adverse?
1.9 prohibits the subsequent representation only if the interests of the new client are materially
adverse to those of the former client.
Did the former client consent?
Under 1.9(a) and (b), a lawyer who would otherwise be disqualified may nevertheless undertake the
representation if the former client gives informed consent, confirmed in writing. Unlike 1.7, 1.9
contains no restriction on the ability of a former client to consent to a subsequent conflict. See also
comment 9.
o Consent v. waiver: the lawyer is required to seek the former clients permission to take on a
new representation; notice to the former client and lack of objection are insufficient.
o Advance consent: consent to conflicts arising in the future is typically used to permit lawyers
to represent one current client in a matter directly adverse to another current client in a matter
that is unrelated to the firms representation of the other client. Such advance consents may
also be used to permit a lawyer to represent a new or an existing client in a matter that will be
adverse to a former client in a substantially related matter.
1.9(b): when lawyers former firm, rather than lawyer personally, represented client in substantially
related matter
When a lawyers former firm, rather than the lawyer himself, represented someone whose interests
are materially adverse to those of the prospective client, the lawyer will be disqualified only if he
actually and individually acquired protected information that would be material to the new matter.
1.9(c): using or disclosing information related to representation of former client
Regulates the use and disclosure of confidential information. This section applies whether or not a
subsequent representation is involved, and it applies even if the lawyers former firmrather than the
individual lawyerrepresented the former client.
Using information: 1.9(c)(1) prohibits a lawyer from using information about a former client except
in ways that would be permitted were the relationship still in effect. The only exception is for
information that has become generally known. The fact that information is in the public record does
not necessarily mean hat the information is generally known within the meaning of 1.9(c) though.
Disclosing information: 1.9(c)(2) prohibits any disclosure of former-client information that would not
be permitted in connection with a current client, regardless of whether the information has become
generally known. See also 1.6.

Rule 1.10: Imputation of Conflicts of Interest: General Rule


1.10(a): Lawyers Associated in a Firm
Imputation imposes each individual lawyer's obligations of client loyalty upon every lawyer with
whom he is associated in a firm. Rule 1.0(c) defines firm or law firm to denote a lawyer
or lawyers in a law partnership, professional corporation, sole proprietorship or other association
authorized to practice law; or lawyers employed in a legal services organization or the legal
department of a corporation or other organization. A government law office is also ordinarily
considered a firm for purposes of the ethics rules, but 1.11 rather than 1.10 regulates the
imputation of conflicts in government law offices. See comment 2 and 1.0 comment 3.
Different kinds of relationships: Imputation is a recurring issue in the relationships discussed below:
of counsel lawyers, temporary lawyers, lawyers who share office space, legal services
organizations, co-counsel, and counsel for co-parties.
Of Counsel Relationships: The practical meaning of of counsel can vary widely among law
firms and even within a firm. As a general matter, of counsel lawyers are treated like firm
partners or associates for conflicts purposes.
Sharing Office Space: Lawyers sharing office space may be considered associated in a firm if
they share information and staff or if they hold themselves out as a firm.
Legal Services Organizations: Whether a legal services organization, or individual components of
it, will be considered a firm for purposes of imputation depends upon the circumstances.
Co-Counsel: When lawyers or law firms serve as co-counsel in a matter, the conflicts of one are
not generally imputed to the other.
Counsel for Co-Parties: Imputation issues also arise out of agreements to cooperate in
representing co-parties.
Exceptions for Nonlawyers: Comment 4 tells us that conflicts of nonlawyers, as well as conflicts
arising from work done before a person became a lawyer, ordinarily will not be imputed to firm
lawyers if the nonlawyer is screened to protect confidential information. (Screened is defined in
Rule 1.0(k) and Comments 810)
No double imputation: Lawyers moving between firms bring with them only the conflicts
created by their actual personal knowledge; they do not bring conflicts based upon knowledge
imputable to them from their association with former firms.
Screening for lateral hires: Rule 1.10(a) remove imputation of a lateral lawyer's disqualification if
the prohibition is based upon Rule 1.9(a) or (b) and arises out of the disqualified lawyer's
association with a prior firm, provided the disqualified lawyer is timely screened and apportioned
no part of the fee. Comment 8 explains that the fee restriction does not prohibit the screened
lawyer from receiving a salary or partnership share established by prior independent agreement,
but the lawyer may not receive compensation directly related to the screened matter. 1.10(a)(2)
(ii) and (iii) list procedural steps required in addition to those contained in the general definition
of screened in Rule 1.0(k), including the content of the prescribed notice to affected former
clients and certifications of compliance.
1.10(b) (Imputation after Conflicted Lawyer Leaves Firm): permits the firm to represent a person
with interests adverse to a client or former client of the departed lawyer as long as the new matter is
not substantially related to that in which the departed lawyer represented the client and no remaining
lawyer has (either in memory or in accessible files) any information protected by Rules 1.6 or 1.9(c)
that is material to the new matter.
1.10(c) (Consent and Waiver): If all affected clients give informed consent under the conditions stated
in 1.7, subsection (c) permits them to waive the imputation of disqualification that would otherwise
be imposed by Rule 1.10(a).
Subsection (d) (Associations with Former or Current Government Lawyers): 1.10 and 1.11, the

disqualification of lawyers associated with former or current government lawyers is governed by


1.11.

1.

2.

3.
4.

Rule 1.11: Special Conflicts of Interest for Former and Current Government Officers and Employees
1.11 provides that except where expressly permitted by law, a lawyer shall not represent a private
client in connection with a matter in which the lawyer participated "personally and substantially" as a
public employee, unless the government consents.
It is sometimes proper, but only if the attorney is screened from the case, receives no fee from it, and
notifies the government. RPC 1.11(b).
1.11(b) permits representation only if the disqualified lawyer is screened from participation and gets
no part of the fee, and if written notice is given to the government agency.
1.11: lawyers formerly employed by the government are not subject to 1.9. Rather 1.11(a), (b), (c),
and (e) replace 1.9(a) and (b). Lawyers currently employed by the government must comply with 1.9,
which is expressly incorporated by 1.11(d). Additionally, 1.11(c) prohibits a lawyer currently or
formerly employed by the government from representing a private client if the lawyer possesses
confidential government information that is damaging to someone with adverse interests.
Moving from one government agency to another: when a lawyer moves from the government to
private practice or from one government agency to another, it may be appropriate according to
comment 5, to treat that second agency as another client for purposes of this rule, as when a lawyer
is employed by a city and subsequently is employed by a federal agency. Similarly, comment 4
notes that successive clients [may be] a government agency and another client, public or private.
However, 1.11(c), addresses subsequent (or concurrent) representation of private clients only.
Similarly, 1.11(d) is phrased in terms of matters in which they participated while in private practice
or nongovernmental employment. Whether two government agencies should be regarded as the
same or different clients is beyond the scope of these rules according to comment 5. But see 1.13.
1.11(a) (participation v. representation): provides that unless the appropriate government agency
consents, a lawyer who has been a public officer or employee is disqualified from representing any
client, whether private or governmental, in whose same matter the lawyer participated personally and
substantially as a public officer or employee. Unlike 1.9, 1.11 applies to lawyers who have served as
public officers or employees of any kind, whether or not in a lawyer capacity.
Public defenders: usually government employees appointed by a judge to represent a private
individual who is being prosecuted by fellow government employees. Because of a public defenders
job working with private individuals, rule 1.9 analyzed successive-employment conflicts in this
context. Thus, such a lawyer may not use screening to avoid imputation of his conflicts.
Personal and substantial: defined as participation through decision, approval, disapproval,
recommendation, the rendering of advice, investigation or otherwise.
Consent by government agency: a former public officers conflict of interest may be waived by the
appropriate government agency if the relevant regulations permit the agency to consent. See 1.7
regarding ability of public entities to waive a conflict.
1.11(b) (screening): a conflict that results from an individually disqualifies lawyers service as a
public officer or employee will not be imputed to his or her firm colleagues if the lawyer is timely
screened and the appropriate government agency is notified in writing. 1.0(k) defines screening as
the isolation of a lawyer from any participation in a matter through the timely imposition of
procedures within a firm that are reasonably adequate under the circumstances to protect information
that the isolated lawyer is obligated to protect under these rules or other law. See 1.0 comment 9 and
10 for explanation of isolation procedures.
1.11(c) (confidential government information): provides that a lawyer who acquired confidential
government information about a person while a public officer may not thereafter represent a private

client whose interests are adverse to that person in a matter in which the information could be used
to the material disadvantage of that person. Confidential government information is specially
defined as information obtained under governmental authority and whichthe government is
prohibited by law from disclosing to the public. Such confidential information is different from
information relating to the representation of a client which is protected by 1.6.
Knowledge not imputed: operated when the lawyer has actual knowledge of the information; the
rule does not operate with respect to information that merely could be imputed to the lawyer (see
comment 8).
No waiver: unlike a conflict resulting from participation in a matter, a conflict resulting from
possession of confidential government information cannot be waived.
1.11(d) (lawyers currently employed by government): lawyers currently serving as public officers or
employees are expressly made subject to the general purpose rules on former-client and current-client
conflicts (1.9 and 1.7, respectively)
Prior nongovernmental participation: a lawyer serving as a public officer may not participate in a
matter in which he participated personally and substantially while in private practice or
nongovernmental employment, unless the appropriate government agency consents. The
prohibition is designed to prevent a lawyer from exploiting public office for the advantage of
another client (comment 3). Note: it is the current government employer, rather than the former
private client, whose consent 1.11(d)(2)(i) requires; the former clients consent is protected by
1.11(d)(1)s incorporation of 1.9.
Concurrently representing private party: a lawyer representing a government agency may
jointly represent a private party if permitted by 1.7 and not otherwise prohibited by law
(comment 9).
No imputation, but screening prudent: comment 2 notes that even though there is no imputation
of the lawyers disqualification to his governmental colleagues, ordinarily it will be prudent to
screen such lawyers.
Negotiating for private employment: 1.11(d)(2)(ii) prohibits a lawyer working for the government
from negotiating for private employment with anyone involved as a party or a lawyer for a party
in a matter in which the lawyer is participating personally and substantially. It represents a
compromise protecting the government lawyers marketability without sacrificing the
governments interest in loyal service.
1.11(e) (matter): a matter is a particular matter involving a specific party. Note: this definition
excludes legislation, rulemaking, and other policy determinations.

Rule 1.13: Organization as a Client


1.13(a): Organization is client
Lawyer represents corporation: unless a lawyer has also formed a lawyer-client relationship with
a constituent, 1.13 clarifies that a lawyer employed or retained by an organization represents the
organization itself, not the individual constituents who act for it. Even if an organizations lawyer
does not represent a particular constituent, the lawyer may have authority under 4.2 to prevent
another lawyer from communicating with the constituent about specific matters.
Closely held corporations: often look and feel quite distinct from public corporations, the entityrepresentation rule usually applies to closely held corporations just as it does to public
corporations. Be on the lookout for situations in which the entitys lawyer inadvertently becomes
an individuals lawyer as well.
Corporate families: by representing one entity the lawyer does not thereby become the lawyer for
affiliated entities. See 1.7, comment 34.
Organizations other than corporations:
Partnerships: a lawyer for a partnership does not automatically represent the partner individually.
But as with counsel for closely held corporations, counsel for the partnership can easily become
an individual partners lawyer as well, even unintentionally.
Governmental organizations: as comment 9 notes, depending on the circumstances, the client may
be a specific agency, a branch of the government, or the government as a whole. Ultimately,
the question is one of law.
Other types of associations: lawyers for other types of entities do not necessarily represent the
constituents.
Confidentiality issues between organization and constituent: duty of confidentiality typically runs to
the organization itself rather than to any of its constituents. But a lawyer may be prohibited from
sharing confidential information with an entity-employer if that information obtained from
constituent who reasonable appeared to be consulting the lawyer as present or prospective client and
the lawyer failed to warn the person that he could act against the persons interests if necessary
because the corporation is his client. See restatement 131. As a general matter, the corporate
attorney-client privilege similarly belongs to the corporation, not it s constituents.
In-house lawyers: Lawyers who are also employees or shareholders
Such attorneys are not only an entitys lawyer by also its employee, and possible its shareholder as
well. When an in-house lawyer asserts a claim against the entity, the ethical constraints attending the
lawyers role as counselparticularly the duty of confidentialitycan make the claim difficult to
pursue.
When constituents conduct likely to harm organization: Under Rule 1.13(b), when a lawyer for an
organization knows that a constituent of the organization is engaged in improper conduct that is
likely to result in substantial harm to the organization, the lawyer must proceed as is reasonably
necessary in the best interest of the organization. See Model Rule 1.0(f) (defining knows as
denoting actual knowledge of the fact in question that may be inferred from circumstances);
Model Rule 1.13, cmt. [[3] (knowledge can be inferred from circumstances, and a lawyer cannot
ignore the obvious). Rule 1.13(b) is, in a sense, a specific application of the lawyer's duties of
competence, diligence, and communication to the organizational context.
Climbing the corporate ladder: unless a lawyer reasonably believes that the organizations best
interests do not so require, the lawyer must report misconduct up the ladder to higher
authorities in the organization, including, if necessary, th highest authority that can act on behalf
of the organization under applicable law. (1.13(b)). In a private organization, the highest
authority will ordinarily be the corporations board of directors or similar governing body.

(Comment 5) This rule requires a lawyer to climb the corporate ladder when the lawyer knows
of a violation of a legal duty reasonably likely to result in substantial corporate injury.
1.13(c): Reporting out
A lawyer is permitted to reveal information relating to the representation outside of the
organization is the lawyer:
o Goes up the organizational ladder and informs the organizations highest authority of
misconduct that is clearly a violation of law reasonably certain to result in substantial
injury to the organization, and
o The highest authority nevertheless fails to address the problem in a timely and
appropriate manner
NOTE: a lawyer is not permitted to disclose confidential information to prevent,
rectify, or mitigate injury, unless the lawyers services are being used to further
the particular crime or fraud.
1.13(d): lawyer investigating or defending claim arising out of corporate wrongdoing
Limits the reporting out authority contained in 1.13(c) if the lawyer has been retained to
investigate an alleged violation of law by the organization, or to defend the organization or a
constituent against claims arising out of an alleged violation of the law, the lawyer does not have
the option of reporting out.
1.13(e): lawyers continuing obligations upon discharge or withdrawal under circumstances governed
by subsections (b) and (c).
If a lawyer takes corrective action pursuant to subsections (b) or (c) and as a result is discharged or
withdraws, 1.13(e) requires the lawyer to take reasonably necessary steps to notify the organizations
highest authority. This rule only requires reasonable steps to assure that the entitys highest authority
is informed of the lawyers discharge and withdrawal and is silent about whether the lawyer is also
permitted to disclose circumstances under which the withdrawal or termination occurred. See
comment 8.
1.13(f): lawyer must clarify role if constituents interests adverse to those of organization
To protect constituents and the organization from problems that could result from confusion about the
lawyers role, 1.13(f) requires the lawyer to clarify the identity of the client when the lawyer knows or
reasonably should know that the organizations interests are adverse to those of a constituent. It
should ordinarily be read together with 4.3 for a fuller understanding of potential Upjohn warnings
in this context.
Failure to clarify rule may result in lawyer-client relationship with constituent: when the lawyer does
not clarify the nature of his or her role in representing the organization, a constituent may conclude
that the lawyer represents the constituent as well as the organization. If this belief is reasonable, it
may give rise to a lawyer-client relationship between the lawyer and the constituent.
1.13(g) (multiple representation): permits a lawyer to represent both an organization and one or more
of its constituents, subject to 1.7 provisions governing conflicts of interest.

1.14 Client with Diminished Capacity


Step 1Write: 1.14 addresses a lawyers ethical obligations when a clients capacity to make
adequately considered decisions in connection with a representation is diminished.

1.14(a) When a clients capacity to make adequately considered decisions in connection with a
representation is diminished, whether because of minority, mental impairment or for some other
reason, the lawyer shall, as far as possible, maintain a normal-client relationship with the client
Normal client-lawyer relationship: requires a lawyer to maintain, as far as possible, a normal client
lawyer relationship with a client with diminished capacity. The rule recognizes that although a
severely incapacitated client may have no power to make legally binding decisions, in many cases,
a client with diminished capacity will have the ability to understand, deliberate upon, and reach
conclusions about matters affecting the clients own well-being. Accordingly, a lawyer representing
a client with diminished capacity is requires, to the extent reasonably possible, to communicate
with that client, work with him, and comply with his objectives concerning the representation.
Representation of minor: explicitly applies to the representation of those whose decision-making
ability is diminished because of minority. Although children may not have the power to make
legally binding decisions, their opinions are generally entitles to some weight in legal proceedings. A
lawyer is not necessarily required to follow the instructions of a minor client if the lawyer believes
that doing so would not be in the clients best interests. However, the lawyers precise obligations in
such circumstances will depend upon a variety of factory including the clients level of maturity, the
nature of the legal matter, the specific facts, and the particular jurisdiction involved.
When client already has legal representative: as noted in comment 4, when a legal representative has
already been appointed for the client, the lawyer should ordinarily look to the representative to make
decisions on the clients behalf.
1.14(b) When the lawyer reasonably believes that the client has diminished capacity, is at risk of
substantial physical, financial, or other harm unless action is taken and cannot adequately act in the
clients own interest, the lawyer may take reasonably necessary protective action, including
consulting with individuals or entities that have the ability to take action to protect the client and, in
appropriate cases, seeking the appointment of a guardian ad litem, conservator, or guardian.
Assessing clients capacity: to determine what, if any, protective action is appropriate, the lawyer
must first assess the clients capacity. Comment 6 suggests that a lawyer consider and balance factors
such as: (1) the clients ability to articulate reasoning leading to a decision; (2) the substantive
fairness of a decision; and (3) the consistency of a decision with the known long-term commitments
and values of the client. If necessary, the lawyer may seek the aid of others to assess the clients
capacity.
Client with poor judgment: a clients poor judgment does not suffice to warrant protective
judgment.
Spectrum of protective measures: 1.14(b) envisions a spectrum of protective actions, from consulting
with individuals who have the ability to take protective action to seeking appointment of legal
representation or guardian. Comment 5 suggests several protective measures that a lawyer can
consider including: (1) consulting family members, (2) using a reconsideration period to permit
clarification or improvement of the circumstances surrounding the clients incapacity; (3) using
voluntary surrogate decision-making tools such as powers of attorney, or (4) consulting support
groups, or professional services.
Seeking appointment of guardian: permits a lawyer for a client with diminished capacity to seek
appointment of a guardian to protect the clients interests if there is no less drastic alternative.
However, a lawyer should not seek to be appointed the clients guardian, except in the most exigent of
circumstances, that is, where immediate and irreparable harm will result from the slightest delay, and

even then, only on a temporary basis. Similarly, a lawyer normally should not represent a third party
petitioning for guardianship over the lawyers client.
Withdrawal from representation: a lawyer who concludes that representing a client with diminished
capacity has become unreasonably difficult may seek to withdraw, if this can be done without
prejudice, but withdrawal is not generally favored.
1.14(c) Information relating to the representation of a client with diminished capacity is protected by
Rule 1.6. When taking protective action pursuant to paragraph (b), the lawyer is impliedly authorized
under Rule 1.6(a) to reveal information about the client, but only to the extent reasonably necessary to
protect the clients interests.
Lawyers implied authorization to reveal information: makes clear that a lawyer must protect the
confidences of a client with diminished decision-making capacity, but that when taking protective
action pursuant to subsection (b), the lawyer is impliedly authorized under 1.6(a) to disclose
information about the clientbut only to the extent reasonably necessary to protect the clients
interests. This provisions is in accord with earlier interpretations of lawyer confidentiality
obligations.
Suicidal Client: as to whether a lawyer may disclose a clients intent to commit suicide, 1.14 permits
a lawyer to disclose clients threat to commit suicide because the overriding social concern for the
preservation of human life dictates that a lawyer may, and even should, take reasonable steps to
preserve the life and well-being if his client and others.
Criminal proceedings: the criminal prosecution of a defendant who is not competent to stand trial is
barred by the due process clause of the 14th amendment.
Emergency legal assistance: comment 9 explains that a lawyer may take emergency protective legal
action when the individuals health, safety, or financial interests are threatened with imminent and
irreparable harm, if the individualor another acting in good faithhas consulted with the lawyer
and the lawyer reasonably believes that the individual has no other lawyer, agent, or other
representative available.

Rule 1.15 Safekeeping Property


1.15: imposes obligations of safekeeping, accounting, and delivery when a lawyer comes into
possession of someone elses money or property. 1.15 is generally applied only when a lawyer gets
possession of anothers property in connection with the representation of a client

a. [1] A lawyer shall [2] hold property of


clients or third persons [3] that is in a
lawyers possession in connection with [4] a
representation separate from the lawyers
own property. [5] Funds shall be kept in a
separate account [6] maintained in the state
where the lawyers office is situated, or
elsewhere with the consent of the client or
third person. [7] Other property shall be
identified as such and [8] appropriately
safeguarded. [9] Complete records of such
account funds and other property [10] shall
be kept by the lawyer and [11] shall be
preserved for a period of [five years] after
termination of the representation.

b.
c.
d.

e.

Anticommingling rule: requires a lawyer to keep the


property of others separate from the lawyers own
property. At the same time, the prohibition also
prevents lawyers from shielding personal assets
from their own creditors by hiding funds in client
trust accounts.
Client funds: misappropriation of client funds usually is
an obvious violation of the rule and is dealt with by
severe disciplinary sanctions.
Stealing from firm: a court may apply 1.15 in addition
to 8.4 to a lawyer who misuses money that belongs
to a law firm
Administration of estates and trusts: 1.15 also applies to
a lawyers misuse of money when administrating
estates and trusts.
Safekeeping of personal property: the lawyer is
responsible for safekeeping property, whether money
or personal property, including documents.
Recordkeeping: requires complete records of funds
held in connection with representation of clients.
[1] A lawyer may deposit [2] the lawyer's own funds in a client trust account [3] for the sole purpose of
paying bank service charges on that account, [4] but only in an amount necessary for that purpose.
[1] A lawyer shall deposit into a client trust account [2] legal fees and expenses that have been paid in
advance, [3] to be withdrawn by the lawyer [4] only as fees are earned or expenses incurred.
[1] Upon receiving funds or other property in which a client or third person has an interest, [2] a
lawyer shall promptly notify the client or third person. [3] Except as stated in this rule or otherwise
permitted by law or by agreement with the client, [4] a lawyer shall promptly deliver to the client or
third person any funds or other property [5] that the client or third person is entitled to receive [6] and,
upon request by the client or third person, shall promptly render a full accounting regarding such
property.
[1] When in the course of
When lawyer and client both have claims: if the lawyer and
representation [2] a lawyer is in
client disagree about hot to divide the funds, the lawyer
possession of property in which two
must distribute any undisputed portion and hold the
or more persons (one of whom may
disputed funds in trust until the matter is resolved.
be the lawyer) claim interests, [3] the Duty to protect claims of clients creditors: third parties, such
property shall be kept separate by the
as creditors of the client, may have valid claims against
lawyer until the dispute is resolved.
client funds or property held by a lawyer. The lawyer may
[4] The lawyer shall promptly
have a duty under applicable law to protect these claims
distribute all portions of the property
against wrongful interference by the client and to refuse to
as to which the interests are not in
surrender the money or property to the client.
dispute.

Rule 1.16: Declining or Terminating Representation


Rule 1.16 provides that an attorney may withdraw from representing a client only under certain
circumstances. Courts have broad discretion in such a decision and will deny permission to withdraw
if it would hurt opposing parties or interfere with the administration of justice. This is so even in
criminal cases, as defendants do not have a Sixth Amendment right to counsel of their choice.
Under some circumstances, lawyers must withdraw from representation and terminate the lawyerclient relationship. In other instances, a lawyer has the option but is not required to withdraw. There
are three types of withdrawal:
Mandatory withdrawal (for violation of rules or law, incapacity, or discharge)
Permissive good cause withdrawal (such as client past crime or fraud, failure to pay, or interference
with representation)
Other permissive withdrawal (where withdrawal would have no materially adverse effect on the
client's interest)
You should also know that all three types of withdrawal are subject to the following limitations:
If the attorney is of record in litigation, court approval is necessary for withdrawal.
The attorney must protect the client's interest on withdrawal by taking "reasonably practicable" steps,
such as giving notice and returning property and papers. What constitutes reasonable notice will
depend on the circumstances, of course.
Certainly you are likely to have gained more confidential information in the process of representing a
client and you do have a duty to preserve those confidences even after withdrawal; however,
sometimes attorneys gain confidential information in the process of screening clients. Attorneys do
have a duty to maintain those confidences, even if they reject the representation.

Must the lawyer withdraw from representation? (Mandatory withdrawal)


Must withdraw if: client has fired you; or you have discovered a conflict of interest between two of
your clients
1.16(a)(1): Will continued representation violate the model rules or some other law?
o Rule 1.16(a)(1) specifies that, unless a court requires the lawyer to continue, a lawyer must
withdraw if the representation will result in violation of the Model Rules or other law. For
example, a lawyer must seek to withdraw if the client is a criminal defendant who insists on
taking the stand to commit perjury.
o While the agreement was based on a past fraud, to continue the representation before the court
would be to assist the client in a fraud on the court as well. The withdrawal is mandatory even if
it will have a material adverse effect on the client's interests. Model Rule 1.16(a)(1)
o You likely had good cause for permissive withdrawal before the complaint was filed (based on
the representation having been made "unreasonably difficult by the client") but you still might
have chosen to continue the representation. However, after the client filed a complaint against
you, you now have a conflict of interest with your client. To continue to represent the client
would be to violate a rule of professional conduct (Rule 1.7) and so you must withdraw. Rule
1.16(a)(1).
1.16(a)(2): Does the lawyers physical or mental health materially impair his ability to represent the
client?
o 1.16(a)(3): Has the lawyer been discharged? A client has an absolute right to discharge a lawyer.
When the client discharges a lawyer, the lawyer must withdraw from the representation.
o 1.16(a)(1): Does the lawyer know the clients conduct is criminal or fraudulent? A lawyer is

required to withdraw when the lawyer knows that the client is using the lawyers services to
perpetrate crimes or frauds.
May the lawyer withdraw from representation? (Permissive withdrawal)
1.16(b): Permits a lawyer to withdraw for one of the enumerated reasons or "if withdrawal can be
accomplished without material adverse effect on the interests of the client."
o May withdraw but only if client isnt hurt if: more lucrative options exist for your time
1.16(b)(2): Does the lawyer reasonably believe the clients conduct is criminal or fraudulent? If the
lawyer is not certain that the lawyers services will result in crimes or frauds, but the lawyer
nevertheless reasonably believes the client is engaging in conduct that is criminal or fraudulent, the
lawyer may withdraw from representation.
o The fact that a client has used your services (negotiating the settlement agreement) to persist in a
cause of action you reasonably believe is criminal or fraudulent does provide you a basis for
permissive withdrawal, even if that withdrawal will cause a material adverse effect on clients
interests. Model Rule 1.16(b)(1)&(2).
1.16(b)(3): Has the client used the lawyers services to perpetuate a crime or fraud? When the lawyer
learns that past services of the lawyer have been used by the client to perpetrate a crime or fraud, the
lawyer may withdraw even if it does not appear that the lawyers current services for the client are
being so used. This rule permits a lawyer to withdraw from a clients representation and distance
himself from the clients crimes or frauds at the earliest possible opportunity to do so.
o May withdraw even if client is hurt if: client has used your services to commit fraud; client is
continuing to use your past services to commit fraud; or clients objectives are repugnant
1.16(b)(4): Does the client insist on taking action that the lawyer considers repugnant or
fundamentally wrong? When, after the lawyer has advised to the contrary, a client intends to continue
with a course of conduct that the lawyer finds morally repugnant or unwise, even though lawful, the
lawyer may withdraw.
o A clients insistence on a repugnant, but not illegal, course of action does provide you a basis for
permissive withdrawal under Model Rule 1.16(b)(4). Where such grounds exist, an attorney may
withdraw even if it will cause a material adverse effect. (Mandatory withdrawal is not justified is
a clients actions are repugnant, but not likely to result in a violation of the rules of professional
conduct or lawModel Rule 1.16(a)(1)).
1.16(b)(5): Has the client substantially failed to meet her responsibilities? If so, has the lawyer
warned that such failure will result in withdrawal? When a client has failed to meet the clients
obligations, most often to pay the lawyers reasonable fee, the lawyer must first warn the client that
the lawyer intends to withdraw if the client does not meet his obligations after the warning, the lawyer
may withdraw.
o Rule 1.16(b)(5) permits withdrawal when "the client fails substantially to fulfill an obligation to
the lawyer regarding the lawyer's services AND HAS BEEN GIVEN REASONABLE WARNING
THAT THE LAWYER WILL WITHDRAW UNLESS THE OBLIGATION IS FULFILLED."
1.16(b)(6): Will the representation result in unreasonable financial burden to the lawyer? This
financial burden is not mere loss, but is on the same order as the sort of financial burden that would
permit a lawyer to decline a court appointment.

Does other good cause for withdrawal exist?


You and your client disagree: While lawyer-client conflicts can provide good cause for
permissive withdrawal, the conflicts must be such that they are interfering with the
representation. Since that is not so here, the attorney must be sure that withdrawal would have no
material adverse effect upon the client's interest. You've been working on a year-long, complex
transaction and you want to back out in the last month. Unless you have co-counsel who can step
in and pick up where you left off without delay or disruption, it would seem that your withdrawal

at this critical stage of the transaction would likely have a material adverse effect. This seems
unlikely.

Is the lawyer prevented from withdrawing?


Will the lawyers withdrawal materially harm the client?
Rule 1.16(d) also provides that the withdrawing attorney must take reasonable steps to protect the
client's interest. These include surrendering papers and property to which the client is entitled
including documents provided by the client to the attorney, evidence and records obtained through
discovery, transcripts of trials and other proceedings, and legal research memoranda relating to the
case. All of these documents would be due to the client. Generally the entire file, including the
attorney's work product (if paid for by the client), must be surrendered, subject to any state laws
allowing retaining liens for unpaid work.
Selecting client:
Apart from instances in which they are appointed to representation, attorneys are generally free to
reject potential clients for any reason at all.
While there are few situations in which an attorney must accept a client, there are several situations in
which an attorney must or should reject representation. Once an individual becomes a client, even if
the attorney later withdraws, the attorney owes duties of competence, confidentiality and conflict-free
representation. Those duties would not ordinarily have arisen had the attorney initially rejected that
representation.
Attorneys should carefully screen clients, then, for potential problems that might later cause the
attorney to be required to withdraw. Besides screening for conflicts of interest, an initial discussion
with a potential client should explore the client's goals, motivations and resources.
Even with careful screening, however, problems arise during representation that may cause an
attorney to seek to withdraw. The next section of the lesson addresses those circumstances and the
relevant legal standards.

1.18: Duties to prospective client


1.18(a): who is a prospective client?
Consultation: anyone who consults a lawyer in good faith about the possibility of forming a lawyerclient relationship is considered a prospective client.
(a) Reasonable expectation of relationship: comment 2 provides that someone who communicates
information unilaterally, without an reasonable expectation that the lawyer is willing to discuss the
possibility of forming a lawyer-client relationship, is not a prospective client protected by 1.18.
(b) Good faith: someone who consults a lawyer as a strategy to preempt that lawyer from representing
someone else does not qualify as a prospective client. See comment 2.
1.18(b): Confidentiality
Confidentiality obligation does not depend upon lawyer-client relationship: codifies the well-settled
proposition that information learned in a consultation is protected whether or not any lawyer-client
relationship ensues. The proposition holds whether the prospective client consults the lawyer
personally or by someone acting on her behalf.
With respect to protecting information, former prospective clients treated same as former clients: puts
prospective clients on par with former (rather than current) clients when it comes to protecting
information. Although a lawyer may neither use not reveal information relating to the representation
of a current client, she may use information relating to the representation of a former client once it is
generally known.
1.18(c): Subsequent adverse representation in substantially related matter
Information that could be significantly harmful: separates the prohibition against using or revealing
information learned in a consultation (the duty of confidentiality) from the prohibition against
subsequent adverse representation in a substantially related matter (the duty of loyalty). Although a
lawyer owes a former prospective client the same duty of confidentiality owed a former fullfledged client, the duty of loyalty is difference. Only if the prospective client revealed information
that could be significantly harmful to him will the lawyer and his firm be prohibited from
representation someone with materially adverse interests in a substantially related matter.
1.18(d)(1): Consent
Even if the lawyer receives disqualifying information under 1.18(c), 1.18(d)(1) permits the
subsequent adverse representation if both affected clients and the prospective client give informed
consent, confirmed in writing. Comment 5 notes that a lawyer may condition a consultation upon the
prospective clients informed consent that nothing disclosed during the consultation will prohibit the
lawyer from representing a different client in the matter, and that if the agreement expressly so
provides, the prospective client may also consent to the lawyers subsequent use of information
received from the prospective client.
1.18(d)(2): Avoiding imputation
Unlike a former client, a former prospective client cannot under 1.18(d)(2) prevent other lawyers in
the firm from undertaking a subsequent adverse representation to which he objects. If the lawyer
minimized the disclosures entailed in the initial consultation, the firm is free to proceed with the
representation as long as it screens the individual lawyer and notifies the former prospective client.

Rule 2.1: Advisor (Related rules: 8.4)


A lawyers role in giving advice often depends on whether he is acting as an advocate or as a
consultant. A lawyer acting as an advocate, for example, may explain the law in a way that favors the
client. Conversely, a lawyer-consultant may need to tell his client that a certain issue is unlikely to be
successful. Rule 2.1 recognizes this role of the lawyer by providing that a lawyer must render his
candid opinion of what the court is likely to do. He must also inform his client of the practical effects
of such a ruling (including not just legal, but also economic, political, social, and moral
consequences). The lawyer and client should discuss the type of legal advice the client is seeking as
provided for in Rule 1.4 also.
Although 2.1 is the model rule that clearly enunciates the lawyers duty to exercise independent
professional judgment in representing a client, rules 1.7 (conflict of interest: current clients), 1.8
(conflict of interest: specific rules), 5.4 (professional independence of a lawyer), 1.9 (duties to former
clients), and 1.18 (duties to prospective clients) address a lawyers independent judgment in a more
nuanced manner.
Insurance defense: independence is of particular concern for lawyers hires by insurance companies to
defend insureds because insurance companies may seek to place restrictions upon the representation.
In general, a lawyer may comply with these restrictions, but only to the extend they do not interfere
with the lawyers independent professional judgment in representing the insured.
Obligations to others: although issues of independence recut in the insurance defense context, the
possibility that a lawyers independent professional judgment may be compromised by allegiances to
others can arise in a variety of other situations too. Frequently, they involve allegiances to people or
entities who are sources of ongoing business for the lawyer. 2.1 may also be invoked in situations
involving lawyers undertaking multiple representations.
Personal interests: 2.1 forbids a lawyers personal interests from affecting his or her independence in
representing a client.
Candid advice: 2.1 requires that the lawyers advice to a client be candid. As comment 1 states, a
client is entitled to straightforward advice expressing the lawyers honest assessment; although legal
advice may involve unpleasant facts and alternatives that a client may be disinclined to confronta
lawyer should not be deterred from giving candid advice by the prospect that the advice will be
unpalatable to the client. Lawyers must not allow their interest in collecting fees for resolving cases
to diminish their candor to clients. 8.4(c) prohibits dishonesty, including to clients).
Advice about nonlegal considerations: 2.1 specifically condone a lawyers reference to nonlegal
considerations (moral, economic, social, political) when giving advice. As a comment explains,
advice given in purely legal terms may be inadequate, particularly when other considerations, such as
cost or the effect upon others, are important to the client.

Rule 3.1: Meritorious Claims and Contentions


Language of the rule: A lawyer shall not bring or defend a proceeding, or assert or controvert an issue
therein, unless there is a basis in law and fact for doing so that is not frivolous, which includes a good
faith argument for an extension, modification or reversal of existing law. A lawyer for the defendant
in a criminal proceeding, or the respondent in a proceeding that could result in incarceration, may
nevertheless so defend the proceeding as to require that every element of the case be established.
Write: A lawyer has a professional obligation to the client, the court, and the lawyers adversaries not
to advance meritless or frivolous arguments.
How do you determine when a claim is frivolous?
Claim must have a basis in law and fact; generally, actions that serve merely to harass or
maliciously injure another
Objectively frivolous claims are often occasioned by intent to harass, embarrass, or otherwise
injure or inconvenience a party, or by some other improper motive
A good-faith argument for an extension, a modification, or a reversal of existing law
A claim or defense is not frivolous merely because the facts have bot first been sully
substantiated or because the lawyer expects to develop vital evidence only by discovery
Similarly, 3.4(d) bars frivolous pretrial discovery requests; 3.2 require reasonable efforts to expedite
litigation.
Rule 3.1 parallels and can be analyzed in tandem with Rule 11 of FRCP. Rule 11 requires, among
other things, that the lawyer representing the party sign a certification that the suit is not intended for
an improper purpose, such as to harass or increase the cost of litigation.
Many states have also adopted anti-SLAPP (strategic litigation against public participation) statutes
that penalize nonmeritorious claims against persons who publically oppose a litigants actions.

Rule 3.2: Expediting Litigation


If an action is not frivolous, does the lawyer violate 3.2 if his client benefits from a delay?
Comment __ provides, The question is whether a competent lawyer acting in good faith would
regard the course of action as having some substantial purpose other than delay. Realizing financial
or other benefit from otherwise improper delay is not a legitimate interest of the client. This
comment seems to suggest that, in determining whether to take an action, the lawyer should not
consider any benefits that flow from such a delay. On the other hand, if an action is non-frivolous,
then is it presumably not improper, and a lawyer may take such action if it would benefit his client.
Further, because many litigation decisions are made for a complicated set of reasons, it is unlikely
that to only reason for a delay is for the client to realize a financial gain. Furthermore, just because a
lawsuit does not win does not mean it is baseless.
Comment __ to Rule 3.2 now states, Although there will be occasions when a lawyer may properly
seek a postponement for personal reasons, it is not proper for a lawyer to routinely fail to expedite
litigation solely for the convenience of the advocates. Thus, lawyers are permitted to ask for delay
to accommodate a personal matter, but they should not routinely delay matters solely on personal
grounds. Although this is not stated in the Comments, a lawyer who seeks delay for a purely personal
reason must first examine whether the delay will affect the client's interests. If such delay may harm
client interests, the lawyer would need to examine Rule 1.1 (Competence), Rule 1.7(a) (Conflicts of
Interest), and Rule 1.4 (duty to keep the client informed). A lawyer who seeks delay, for more than a
few days, based upon a personal reason should consider informing the client about the lawyer's need
for a delay and is obligated to do so per Rule 1.4.
3.2 recognizes that lawyers dilatory tactics may impede the administration of justice, thus burdening
opposing parties and wasting public resources. While 1.3 sets forth the general requirements that
lawyers act with reasonable diligence and promptness in representing a client, 3.2 specifically
required lawyers to attempt to expedite litigation. Note, however, the rule qualifies the duty to
expedite litigation by stating it must be consistent with the interests of the client.
Failing to prosecute or defend claim (preparing, filing, and serving pleadings, motions, and other
court documents): 3.2 requires that a lawyer prosecute or defend a clients clime without undue delay.
This means all necessary filings and service must be made promptly.
Appearing at hearings and obeying court orders: 3.2 also requires appearances at scheduled hearings
and adherence to court orders.
Dilatory discovery practices: barred by 3.2
Delaying disposition: action or inaction that delays a final disposition, even if it causes no harm to the
client can violate 3.2.

Rule 3.3: Candor toward the Tribunal


Tribunal: 3.3 requires candor to a tribunal. 1.0 defines a tribunal as a court, an arbitrator in a
binding arbitration proceeding or a legislative body, administrative agency or other body acting in an
adjudicative capacity. If the tribunals judgment will not be binding, 3.3 does not apply. However, if
the lawyer is representing a client before a legislative body or administrative agency in a
nonadjudicative proceeding, 3.9 require the lawyer to comply with 3.3(a)(c).
Statements made in lawyers personal capacity: 3.3 governs the conduct of a lawyer who is
representing a client in a proceeding. There is an argument that when the lawyer is a client he owes
no such duty to the tribunal.
3.3(a)(1) (statements made in representing client): prohibits a lawyer from knowingly making a
false statement of fact or law to a tribunal, whether material or not, whether orally or in writing,
and whether in an affidavit, a pleading, or other document.
Statements or omissions that mislead: failure to make a disclosure can be the equivalent of an
affirmative misrepresentation. Courts routinely employ 3.3(a)(1) discipline lawyers who have
misled through their silence.
Duty to correct own material false statement: 3.3(a)(1) requires a lawyer to correct a false statement
of material fact or law previously made to the tribunal by the lawyer.
Statements about bar status or representational capacity: a lawyer who misstates his eligibility to
serve as counsel violates 3.3(a)(1).
Notarizing signature: a lawyer who notarizes a signature on a document submitted to the court is
making a statement within the meaning of 3.3(a)(1).
Statements about discovery: misrepresenting the status of discovery or the availability of information
sought in discovery violates 3.3(a)(1).
Statements about judicial officers: false statements concerning judicial officers have been held to
violate 3.3 as well as 8.2.
3.3(a)(3) (failure to disclose legal authority known to be directly adverse to clients position): requires
a lawyer to disclose legal authority in the controlling jurisdiction that the lawyer knows in directly
adverse to the clients position and that has not been presented by opposing counsel.
Offering false evidence: prohibits a lawyer from knowingly offering false evidence. There is a
special exception for a criminal defense lawyer who knows his client intends to testify falsely.
Remedial measures: although the prohibition against offering false evidence is not limited
materially, the duty to remedy is not triggered unless the evidence is material. This duty arises
whether the evidence was offered by the lawyer, the client, or a witness called by the lawyer.
Comment 10 provides that reasonable remedial measures include remonstrating with the client
privately, withdrawing, and whether or not the lawyer withdraws, making such disclosure to the
tribunal as is reasonably necessary to remedy the situation.
Criminal cases (client perjury): 3.3(a)(3) allows a lawyer to refuse to offer evidence that the
lawyer reasonably believesbut does not knowis false. This does not apply to testimony by a
client who is a defendant in a criminal proceeding; a criminal defense lawyer must honor the
clients decision to testify unless, according to comment 9, he knows the testimony will be
false. Comment 7 acknowledges that cross-examining a client whereby he tells his side via
narrative is one imperfect option available in the client perjury dilemma. Known as the
Washington __, this form of cross-examination signals to a judge that a client is perjuring
himself.

Knowledge: the level of knowledge sufficient to trigger the prohibition against presenting a
criminal defense clients false testimony is very high.
3.3(b) (when lawyer knows of criminal or fraudulent conduct relating to proceeding): states that a
lawyer who knows that a person intends to engage, is engaging, or have engages in criminal or
fraudulent conduct related to the proceeding shall take reasonable remedial measures, including, if
necessary, disclosure to the tribunal. This reflects a special obligation on the part of lawyers to
protect a tribunal against criminal or fraudulent conduct that undermines the integrity of the
adjudicative process.
3.3(c) (duration of duty): specifies that the obligation of subsections (a) and (b) continue to the
conclusion of a proceeding.
3.3(d) (ex parte proceedings):

Rule 3.4: Fairness to Opposing Party and Counsel


Rule 3.4 explains the lawyers duties to adverse parties and counsel to ensure that litigation is
conducted fairly. (related: 1.2(d) (may not counsel or assist a client to engage in criminal or
fraudulent conduct) and 8.4(a) (prohibited from assisting or inducing person in violating ethics rules).

[1] A lawyer shall not:


(a) [2] unlawfully [3] obstruct another

party's access to [4] evidence or [5]


unlawfully alter, destroy or conceal [6] a
document or other material [7] having

potential evidentiary value. [8] A lawyer


shall not counsel or assist another

person to do any such act;

(b)

(c)

(d)

(e)

(f)

Obstructing another partys access to evidence: a


lawyer may not unlawfully obstruct another partys
access to evidence.
Procuring absence of witness: constitutes the
obstruction of evidence in violation of 3.4(a).
Unlawful concealment v. legitimate failure to disclose:
does not impose a duty to volunteer all relevant
information that a lawyer has, but prohibits concealing
potential evidence a lawyer has a legal duty to disclose.
[2] falsify evidence, [3] counsel or assist Falsifying evidence: 3.4(b) is owed to an opposing
a witness [4] to testify falsely, or [5]
party and counsel, while 3.3 casts the obligation as part
offer an inducement to a witness [6] that
of a lawyers duty of candor toward the tribunal.
is prohibited by law;
Offering illegal inducement to witness: offering lawful
inducement, however, does not violate the rule.
Witness fees: it is not improper to pay a witnesss
expenses or to compensate an expert witness on terms
permitted by law. See c.3.
[2] knowingly disobey [3] an obligation Court orders: prohibits lawyers from disobeying, or
under the rules of a tribunal, [4] except
advising their clients to disobey court orders.
for an open refusal based on an
Court rules: prohibits lawyers from disobeying the rules
assertion that no valid obligation exists;
of the tribunal
[2] in pretrial procedure, [3] make a (i) General prohibition: lawyers duty of fairness to the
frivolous discovery request or fail to
opposing party and counsel prohibits the lawyer from
make reasonably diligent effort [4] to
abusing pretrial discovery procedures. A lawyer shall
comply with a legally proper discovery
not make a frivolous discovery request or fail to make a
request by [5] an opposing party;
reasonably diligent effort to comply with a legally
proper discovery request by an opposing party.
[2] in trial, [3] allude to any matter that(ii) Improper questions or statements: prohibits lawyer
the lawyer does not reasonably believe
misconduct at trial and limits certain trial tactics. It
is [4] relevant or that will not be
prohibits a lawyer from alluding to irrelevant matters or
supported by admissible evidence, [5]
matters not supported by admissible evidence.
assert personal knowledge of facts in

issue [6] except when testifying as a


witness, or [7] state a personal opinion
as to the justness of a cause, the
credibility of a witness, the culpability
of a civil litigant or the guilt or
innocence of an accused; or
[2] request a person other than a client [3] to refrain from voluntarily giving relevant information to
another party [4] unless:
[5] the person is a relative or an employee or other agent of a client; and
[6] the lawyer reasonably believes that the person's interests will not be adversely affected by

refraining from giving such information.

Rule 3.5: Impartiality and Decorum of the Tribunal

[1] A lawyer shall not:


(a) [2] seek to influence a judge, juror,
Gifts, loans, and contributions: a lawyer may not provide gifts
prospective juror or other official [3]
or loans to a judge or other official in an attempt to
by means prohibited by law;
influence such person. However, gifts that constitute

ordinary social hospitality, such as social dinners, rides,


birthday recognitions, or flowers are generally permissible.

Although contributions to a judges campaign may be


permitted, campaign support intended to influence is a
violation of 3.5
Judicial awards and honors: may implicate 3.5(a) as well as
8.4(e)
(b) [2] communicate ex parte with such a Ex parte communication with judges: prohibits ex parte
person [3] during the proceeding [4]
communication with a judge during the course of a legal
unless authorized to do so by law or
proceeding unless the communication is authorized by law
court order;
or court order. Such communication is barred regardless

of the lawyers motivation for the contact.

Communication authorized by law or court order: 3.5(b)


contains an exception for communication that is
authorizedby law or court order. Court rules and case
law within a particular jurisdiction must be consulted to
determine the legitimacy of an ex parte communication.
When judges initiate ex parte communication: the restriction
on ex parte communication apply equally whether the
judge or the lawyer initiates contact. When the judge
initiates contact, the lawyer has an obligation to refuse to
participate in the communication and to request that
opposing counsel be party to, or notified of, an
communication.
Written submissions to judges: as a rule, copies of all
submissions to a judge must be furnished simultaneously
to opposing counsel
(c) [2] communicate with a juror or

prospective juror after discharge of


the jury if:
(i) [3] the communication is
prohibited by law or court order;
(ii) [3] the juror has made known to
the lawyer a desire not to
communicate; or
(iii) [3] the communication involves
misrepresentation, coercion,
duress or harassment; or
(d) [2] engage in conduct intended to

disrupt a tribunal.

Rule 3.6: Trial Publicity


Restricts a lawyer from making out-of-court statements that the lawyer knows or reasonably should
know will be publicly disseminates and will have a substantial likelihood of materially prejudicing
an adjudicative proceeding in the matter. By its terms, the rule applies not only to lawyers currently
representing clients but also to those who participated in the investigation or litigation of a matter.
Consider:
Was the information revealed available in the public record?
Was there a gag order in place?
How did the timing of the statement affect?

A lawyer who is participating or has participated in the investigation or litigation of a matter shall
not make an extrajudicial statement that the lawyer knows or reasonably should know will be
disseminated by means of public communication and will have a substantial likelihood of
materially prejudicing an adjudicative proceeding in the matter.
Notwithstanding paragraph (a), a lawyer may state:
(1) the claim, offense or defense involved and, except when prohibited by law, the identity of the
persons involved;
(2) information contained in a public record;
(3) that an investigation of a matter is in progress;
(4) the scheduling or result of any step in litigation;
(5) a request for assistance in obtaining evidence and information necessary thereto;
(6) a warning of danger concerning the behavior of a person involved, when there is reason to believe
that there exists the likelihood of substantial harm to an individual or to the public interest; and
(7) in a criminal case, in addition to subparagraphs (1) through (6):

(i) the identity, residence, occupation and family status of the accused;

(ii) if the accused has not been apprehended, information necessary to aid in
apprehension of that person;

(iii) the fact, time and place of arrest; and

(iv) the identity of investigating and arresting officers or agencies and the length
of the investigation.
Notwithstanding paragraph (a), a lawyer may make a statement that a reasonable lawyer would
believe is required to protect a client from the substantial undue prejudicial effect of recent
publicity not initiated by the lawyer or the lawyer's client. A statement made pursuant to this
paragraph shall be limited to such information as is necessary to mitigate the recent adverse
publicity.
No lawyer associated in a firm or government agency with a lawyer subject to paragraph (a) shall
make a statement prohibited by paragraph (a).

Rule 3.8: Special Responsibilities of a Prosecutor


Write: 3.8 imposes special obligations upon prosecutors in criminal cases. As a minister of justice,
a prosecutor has a duty to see that the defendant is accorded procedural justice and that guilt is
decided upon the basis of sufficient evidence.

[1] The prosecutor in a criminal case shall:


(a) [2] refrain from prosecuting a charge that [3] the
Prohibits a prosecutor from bringing a charge
prosecutor knows is [4] not supported by probable
that the prosecutor has actual knowledge is
cause;
unsupported by probable cause.
(b) [2] make reasonable efforts to [3] assure that the accused has been advised of the right to, and the
procedure for obtaining, counsel and [4] has been given reasonable opportunity to obtain counsel;
(c) [2] not seek to obtain [3] from an unrepresented accused [4] a waiver of important pretrial rights, such
as the right to a preliminary hearing;
(d) [2] make timely disclosure [3] to the defense [4] of
The U.S. Constitution requires prosecutors to
all evidence or information known to the prosecutor
provide the defense with any favorable
[5] that tends to negate the guilt of the accused or
evidence that is material to guilt,
mitigates the offense, and, [6] in connection with
punishment, or impeachment.
sentencing, [7] disclose to the defense and to the
Requires disclosure of all information that
tribunal [8] all unprivileged mitigating information
tends to negate the guilt of the accused or
[9] known to the prosecutor, [10] except when the
mitigates the offense.
prosecutor is relieved of this responsibility by a
A prosecutors belief that evidence is not
protective order of the tribunal;
exculpatory does not necessarily shield him
from discipline under the rule.
(e) [2] not subpoena a lawyer [3] in a grand jury or other Limits a prosecutors ability to subpoena other
criminal proceeding [4] to present evidence [5] about
lawyers in grand jury and other criminal
a past or present client [6] unless the prosecutor
proceedings.
reasonably believes:
[7] the information sought is not protected from
disclosure by any applicable privilege;
[8] the evidence sought is essential to the
successful completion of an ongoing
investigation or prosecution; and
[9] there is no other feasible alternative to obtain
the information;
(f) [2] except for statements that are necessary to inform Prohibits a prosecutor from making
the public of the nature and extent of the prosecutor's
extrajudicial comments that have a
action and that serve a legitimate law enforcement
substantial likelihood of heightening public
purpose, [3] refrain from making extrajudicial
condemnation of the accused, except for
comments that have a [4] substantial likelihood of
certain legitimate statements necessary to
heightening public condemnation of the accused and
inform the public of the proceedings. The
[5] exercise reasonable care to prevent investigators,
rule does on to require the prosecutor to
law enforcement personnel, employees or other
exercise reasonable care to prevent
persons assisting or associated with the prosecutor [6]
anyone assisting or associates with the
in a criminal case [7] from making an extrajudicial
prosecutor from making any statements the
statement [8] that the prosecutor would be prohibited
prosecutor could not make under 3.8 or 3.6.
from making under Rule 3.6 or this Rule.
(g) [2] When a prosecutor knows of new, credible and
Creates post-conviction duties for prosecutors
material evidence [3] creating a reasonable likelihood
to disclose new, credible and material

[4] that a convicted defendant [5] did not commit an


evidence they come to know of that
offense of which the defendant was convicted, [6] the
creates a reasonable likelihood that a
prosecutor shall:
convicted defendant did not commit an
offense and to seek to remedy a
[7] promptly disclose that evidence to an
conviction when they come to know of
appropriate court or authority, and
clear and convincing evidence
[8] if the conviction was obtained in the
establishing that a defendant in the
prosecutor's jurisdiction,
prosecutors jurisdiction did not commit
[9] promptly disclose that evidence to the
the offense for which he was convicted.
defendant unless a court authorizes delay,
and
[10] undertake further investigation, or make
reasonable efforts to cause an investigation,
to determine whether the defendant was
convicted of an offense that the defendant did
not commit.
(h) [2] When a prosecutor knows of clear and convincing evidence [3] establishing that a defendant [4] in
the prosecutor's jurisdiction [5] was convicted of an offense [6] that the defendant did not commit, [7]
the prosecutor shall seek to remedy the conviction.

Rule 4.1: Truthfulness in Statements to Others


Related rules: 1.2, 1.6, 3.3, 3.4, 4.1, 8.4
See pp. 741 (and comment 2)
[1] In the course of representing a client [2] a
lawyer shall not knowingly:
(a) [3] make a false statement of material fact
or law to a third person; or

o
o
o

o
o

(b) [3] fail to disclose a material fact [4] when


disclosure is necessary to avoid assisting a
criminal or fraudulent act by a client, [5]
unless disclosure is prohibited by Rule 1.6.

Applies to a lawyers statements and omissions


in the course of representing a client.
Prohibits a lawyer from knowingly making
false statements. According to 1.0(f), this
requires actual knowledge of the fact in
question, which may be inferred from
circumstances.
A statement is material for purposes of 4.1(a) if
it could or would influence the hearer.
Misrepresentations include partially true but
misleading statements or omissions that are the
equivalent to affirmative false statements. c.1
Note: a misrepresentation by omission under
4.1(a) is different from a violation of 4.1(b)s
affirmative obligation to disclose; 4.1(b) comes
into play only if the lawyer would otherwise be
assisting a client in a crime or fraud.
Misrepresentations made in the course of
litigation can violate 4.1(a), 3.3, 3.4, and 8.4.
Certain statements ordinarily are not taken as
statements of material fact under generally
accepted conventions in negotiation, including
estimates of price or value, and a partys
intentions regarding acceptable settlement. c.2
A lawyer must come forward with material
facts if nondisclosure would amount to
assisting in a clients crime or fraud. See 1.2(d)
Requires disclosure of a material fact to avoid
assisting in a clients crime or fraud unless
disclosure is prohibited by 1.6. 1.6 generally
bars lawyers from disclosing any information
relating to the representation of a client but
1.6(b) makes an exception, and permits
disclosure when a client is using the lawyers
services to further certain crimes or frauds.
Thus, 4.1(b) requires disclosure if both 1.6(b)
(2) and (3) are met.

Rule 4.2: Communication with Person Represented by Counsel

[1] In representing a client, [2] a lawyer shall


not communicate about the subject of the
representation [3] with a person the lawyer
knows [4] to be represented by another lawyer
in the matter, [5] unless the lawyer has the
consent of the other lawyer or is authorized to
do so by law or a court order.

Codifies the rule that if a person is represented


in a particular matter, lawyers must go through
his counsel if they are to communicate with
him about it.
No waiver; the client cannot waive protection
of 4.2
A lawyer is permitted to talk to someone who
already has counsel but who wants a second
opinion from a lawyer who is not representing a
party in the matter.
Does not restrict participants in a matter from
communicating with each other directly,
whether or not they have their own lawyers
Comment 4 specifies, a lawyer is not
prohibited from advising a client concerning a
communication that the client is legally entitled
to make.
Does not prohibit a lawyer from
communicating with a represented person about
something other than the subject of the
representation.

How does the lawyer know if someone is represented?


Consent of opposing counsel is required to communicate with a represented person only if a
lawyer knows a person is represented. Comment 8 defines knowledge as actual knowledge,
but explains that it may be inferred from the circumstances.
When an organization is represented, which constituents are deemed represented?
Managing/speaking agent test: prohibits direct contact with an employee who had managerial
responsibility or who could speak for the company.
Under comment 7, the permissibility of informal contacts no longer turns on the constituents
ability to make admissions on the organizations behalf, or on the constituents managerial
responsibility. Instead, it turns on whether the constituent supervises, directs or regularly
consults with the organizations lawyer concerning the matter. Comment 7 also prohibits
communications with a constituent whose act or omission in the matter may be imputed to the
organization.

Rule 4.3: Dealing with Unrepresented Person


In dealing on behalf of a client with a person who is not represented by counsel, a lawyer shall not
state or imply that the lawyer is disinterested. When the lawyer knows or reasonably should know that
the unrepresented person misunderstands the lawyer's role in the matter, the lawyer shall make
reasonable efforts to correct the misunderstanding. The lawyer shall not give legal advice to an
unrepresented person, other than the advice to secure counsel, if the lawyer knows or reasonably
should know that the interests of such a person are or have a reasonable possibility of being in
conflict with the interests of the client.
Attempts to ensure that unrepresented people, whatever their interests might be, are not misled when
dealing with a lawyer who is acting for a client.
4.3 prohibits a lawyer from giving legal advice other than the advice to secure counsel to an
unrepresented person if the persons interests might conflict with those of the lawyers client.
Interviewing an unrepresented person?
Before interviewing unrepresented employee of represented corporation, lawyer and investigator
must (1) fully disclose their representative capacity to the employee, (2) state the reason for seeking
the interview as it concerns the attorney's client and the employer, (3) inform the individual of his or
her right to refuse to be interviewed, (4) inform the person that he or she has the right to have their
own counsel present, and finally (5) may not under any circumstances seek to obtain attorney-client
or work product information from the employee
Communication with employees of a represented organization
Because Rule 4.2 prohibits a lawyer from communicating with a represented person without the
consent of that person's lawyer (unless authorized by law or court order), the fact that counsel
represents a corporation or other entity means that a lawyer may not be able to communicate directly
with some of that entity's employees. This will depend upon the role the employee plays in the
organization and in the matter at issue.
In general, Rule 4.2 applies if the employee meets some combination (depending upon the
jurisdiction) of the following criteria: the employee has managerial responsibilities within the entity,
the employee's actions or omissions could be imputed to the entity, the employee's statements could
bind the entity, the employee's admissions would be admissible against the entity as an exception to
the hearsay rule, and/or the employee is responsible for implementing the advice of the entity's
lawyers. For discussion see the Annotations to Model Rule 4.2 (Communication with Person
Represented by Counsel) and Model Rule 1.13 (Organization as Client).
Employees who do not fit into one of these categories are, like former employees, not deemed to be
represented by the corporation's lawyer; communications with them are therefore governed by Rule
4.3 rather than Rule 4.2.
To avoid a misunderstanding, according to Comment [1], a lawyer will typically need to identify
the lawyer's client and, where necessary, explain that the client has interests opposed to those of the
unrepresented person. Many courts impose considerably more specific requirements when it comes
to interviewing corporate employees and, for example, require the lawyer to advise the employees of
their right to refuse to be interviewed and their right to have counsel present.

Rule 5.1: Responsibilities of Partners, Managers, and Supervisory Lawyers


Rule 5.1 imposes a duty upon partners, lawyers with comparable managerial authority, and lawyers
who directly supervise other lawyers to oversee the conduct of lawyers within their firm or
organization. The rule establishes the principle of supervisory responsibility without introducing
vicarious liability.

(a) [1] A partner in a law firm, and a lawyer who


individually or together with other lawyers
possesses comparable managerial authority in a
law firm, [2] shall make reasonable efforts to
ensure that [3] the firm has in effect measures
[4] giving reasonable assurance that [5] all
lawyers in the firm conform to the Rules of
Professional Conduct.

A lawyer who is a partner or manager in a firm


must make reasonable efforts to establish
measures ensuring that lawyers within the firm
comply with the Model Rules, including; (1)
procedures for detecting and resolving conflicts
of interest, (2) ensuring compliance with
deadlines, (3) safeguarding client funds and
property, and (4) supervising inexperienced
lawyers.
See also comment 1 & 2.
(b) [1] A lawyer having direct supervisory authority A lawyer with direct supervisory responsibility
over another lawyer [2] shall make reasonable
over another lawyer has an obligation to make
efforts to [3] ensure that the other lawyer
reasonable efforts to ensure the other lawyer
conforms to the Rules of Professional Conduct.
complies with the Model Rules.
Assigning excessive caseloads to subordinates may
violate 5.1(b)
Applies to supervision of outsourced lawyers not
directly affiliated with the supervising lawyers
firm
(c) [1] A lawyer shall be responsible for [2] another Any lawyer may be held responsible in discipline
lawyer's violation of the Rules of Professional
for another lawyers ethical violations if the
Conduct if:
lawyer orders or ratifies the specific conduct
(1) [3] the lawyer orders or, with knowledge of
involved.
the specific conduct, ratifies the conduct
If the lawyer is a partner, manager, or direct
involved; or
supervisor of the offending lawyer, he may be
(2) [3] the lawyer is a partner or has comparable
answerable in discipline for the misconduct
managerial authority in the law firm in
merely by having knowledge of it at a time
which the other lawyer practices, or has
when its consequences can be avoided or
direct supervisory authority over the other
mitigated and failing to take reasonable
lawyer, and [4] knows of the conduct at a
remedial action.
time when its consequences can be avoided
or mitigated [5] but fails to take reasonable
remedial action.

Rule 5.2: Responsibilities of a Subordinate Lawyer


Rule 5.2 provides that a lawyer is not excused from the duty to act ethically simply because actions
are taken at the direction of another. However, a subordinate lawyer is not subject to discipline for
acting in accordance with a supervisory lawyer's reasonable resolution of an arguable question of
professional duty.

(a) [1] A lawyer is bound by the Rules of


Professional Conduct [2] notwithstanding that
the lawyer acted at the direction of another
person.
(b) [1] A subordinate lawyer does not violate the
Rules of Professional Conduct if [2] that
lawyer acts in accordance with a supervisory
lawyer's [3] reasonable resolution of [4] an
arguable question of professional duty.

(a)

(b)
(c)

(d)

5.4 Professional independence of a lawyer (related rules: 1.8, 2.1)


Intended to protect a lawyers independent professional judgment by limiting the influence of
nonlawyers on the lawyer-client relationship.
[1] A lawyer or law firm shall not share legal fees
(a) Prohibits a lawyer or law firm from paying a
with a nonlawyer, [2] except that:
nonlawyer a portion of the legal fees generated
(1) an agreement by a lawyer with the lawyer's
by any particular matter unless one of the four
firm, partner, or associate [3] may provide for
narrow exceptions apply (5.4(a)(14)).
the payment of money, over a reasonable
(b) Paying to recruit clients: a lawyer who shares
period of time [4] after the lawyer's death, [5]
legal fees in return for referrals violates 5.4(a).
to the lawyer's estate or to one or more
(c) Using a third person to sign up clients is also
specified persons;
addressed by 7.2(b) and 8.4(a)
(2) a lawyer [3] who purchases the practice of a
(d) 5.4(a)(1) permits a lawyer and a firm to agree
deceased, disabled, or disappeared lawyer [4]
that upon the lawyers death, the firm will make
may, pursuant to the provisions of Rule 1.17,
payments to the lawyers estate
[5] pay to the estate or other representative of
(e) 5.4(a)(3) permits a lawyer or law firm to include
that lawyer the [6] agreed-upon purchase price;
nonlawyer employees in profit-sharing
(3) a lawyer or law firm [2] may include
compensation and retirement plans, but only if
nonlawyer employees in a compensation or
the profits being shared are not tied to particular
retirement plan, [3] even though the plan is
clients or particular matters.
based in whole or in part on a profit-sharing
(f) 5.4(a)(4) permits a lawyer to share courtarrangement; &
awarded legal fees with a nonprofit organization
(4) a lawyer may share [2] court-awarded legal
that employs, retains, or recommends the lawyer.
fees [3] with a nonprofit organization [4] that
employed, retained or recommended
employment of the lawyer in the matter.
[1] A lawyer shall not form a partnership with [2] a (a) Prohibits a lawyer from entering a partnership
nonlawyer if [3] any of the activities of the
with a nonlawyer if any of the partnerships
partnership consist of the practice of law.
activities constitutes the practice of law.
[1] A lawyer shall not permit [2] a person who
(b) Prohibits a lawyer from allowing a person who
recommends, employs, or pays [3] the lawyer to
recommends, employs, or pay the lawyer to
render legal services for another [4] to direct or
interfere with the lawyers exercise of
regulate the lawyer's professional judgment in
independent professional judgment on behalf of
rendering such legal services
a client.
(c) Complements 1.8(f)(2), which prohibits a lawyer
from accepting payment from a third party if it
would interfere with the lawyers independent
professional judgment.
[1] A lawyer shall not [2] practice with or in the
(1) Restates 5.4(c) and 2.1 that a lawyer exercise
form of [3] a professional corporation or association
independent professional judgment on a clients
authorized to practice law [4] for a profit, if:
behalf.
[5] a nonlawyer owns any interest therein, [6]
except that a fiduciary representative of the
estate of a lawyer may hold the stock or
interest of the lawyer for a reasonable time
during administration;
[5] a nonlawyer is a corporate director or
officer thereof or [6] occupies the position of
similar responsibility in any form of association
other than a corporation; or
[5] a nonlawyer has the right to direct or
control the professional judgment of a lawyer.

Rule 6.2 Accepting Appointments


A lawyer shall not seek to avoid appointment by a tribunal to represent a person except for good
cause, such as:
(a) representing the client likely to result in a violation of the rules or law; or
Incompetent: While your license to practice allows you to practice all areas of law, that does not
mean you will be competent to do so, especially in a complicated case such as this. Attorneys can
seek to be excused from appointment on the grounds of competence. See Easley v. State, 334 So.
2d 630 (Fla. Dist. Ct. App. 1976)(counsel cannot be held in contempt for attempting to withdraw
as appointed counsel for defendant on grounds of incompetence.) Remember, however, that one
can become competent in many instances with appropriate preparation and courts will not excuse
an attorney simply because he or she is unwilling, rather than unable, to put in the required
preparation. See Reese v. Owens-Corning Fiberglass, 962 F. Supp. 1418 (D. Kan. 1997).
(b) representing the client is likely to result in an unreasonable financial burden on the lawyer; or
The standard is not "negative financial impact" but "unreasonable financial burden" and the
comments to Rule 6.2 and the cases interpreting the rule indicate that the burden must involve a
"financial sacrifice so great as to be unjust."
(c) the client or the cause is so repugnant to the lawyer as to be likely to impair the attorney-client
relationship or the lawyer's ability to represent the client.
Rule 6.2: Accepting appointments
If you had been appointed to represent a particular client, you would have an obligation not to avoid
that appointment unless you had good cause. While repugnance of the client or cause can constitute
good cause, comment 2 to Rule 6.2 indicates that repugnance should be "likely to impair the attorneyclient relationship or the lawyer's ability to represent the client."
A client you find repugnant (see also 1.16)
Under these circumstances there is no legal obligation. There is nothing in the text of the Model
Rules that requires representation of a particular client. There surely is, however, an ethical
obligation, as part of an attorney's general obligations to render public interest legal service.

7.1 Communications concerning a lawyers services


Although Rule 7.1 is most often implicated in the context of lawyer advertising, the rule is not limited
to advertisements. It applies to all communications made by a lawyer and the lawyer's agents.
The U.S. Supreme Court has held that the First Amendment protects commercial speech, which
includes lawyer advertising.
[1] A lawyer shall not [2] make a
false or misleading
communication [3] about the
lawyer or the lawyer's services.
[4] A communication is false or
misleading if it [5] contains a
material misrepresentation of
fact or law, or [6] omits a fact
necessary [7] to make the
statement considered as a whole
[8] not materially misleading.

This prohibition includes both explicit falsehoods and


falsehoods implicit in other claims.
Rule 7.1 prohibits lawyers from making statements that are
literally true, but misleading. Model Rule 7.1, cmt. [2]. This
may involve a lawyer omitting important information and/or
providing information in a way that leads another to reach an
unwarranted conclusion about the lawyer, the lawyer's
services, or the results the lawyer could obtain for a client.
Lawyers advertising legal fees must disclose information
about clients' responsibilities to pay costs.
Omitting or misrepresenting facts about the qualifications of
individuals listed on firm letterhead and business cards may
also violate Rule 7.1.
As Comment [3] notes, an otherwise true statement may be
misleading if presented in a way that would lead a reasonable
person to form an unjustified expectation that the lawyer can
obtain the same results for that person as the lawyer obtained
in previous matters, without reference to the specific factual
and legal circumstances of each client's case.
The advertising that has been permitted under Rule 7.1
includes statements that convey verifiable factual information
and those that are more generally descriptive but not
inherently misleading.

7.2 Advertising

(a) [1] Subject to the requirements of Rules 7.1 and 7.3,


[2] a lawyer may advertise services [3] through
written, recorded or electronic communication,
including public media.

(b) [1] A lawyer shall not give anything of value to a

person [2] for recommending the lawyer's services


[3] except that a lawyer may
(1) [4] pay the reasonable costs of advertisements or
communications permitted by this Rule;

(2) [4] pay the usual charges of a legal service plan


or a not-for-profit or qualified lawyer referral
service. [5] A qualified lawyer referral service
is a lawyer referral service that [6] has been
approved by an appropriate regulatory authority;
(3) [4] pay for a law practice in accordance with
Rule 1.17; and
(4) [4] refer clients to another lawyer or a
nonlawyer professional pursuant to [5] an
agreement not otherwise prohibited under these
Rules that [6] provides for the other person to
refer clients or customers to the lawyer, if
(g) [7] the reciprocal referral agreement is not
exclusive, and
i. [8] the client is informed of the existence
and [9] nature of the agreement.
(c) Any communication made pursuant to this Rule

shall include the name and office address of at least


one lawyer or law firm responsible for its content.

This rule makes clear that lawyers are free


to advertise their services through various
media.
Written communications: advertising in
print media is constitutionally protected if
it is not false or misleading.
Recorded communications: includes
prerecorded telephone messages; if
however, the lawyer knows that the person
receiving the call does not want to be
contacted, the call constitutes solicitation
and is barred by 7.3.
A lawyer may not pay someone else to
recommend his services unless excepted
under 7.2(b). This rule also prohibits
indirect compensation for referrals.
Reasonable costs of advertising: see c.5.
Referrals pursuant to agreement: allows
mutual referrals of prospective clients
between a lawyer and another lawyer or
nonlawyer professional if otherwise
permitted by the rules and (1) the
agreement is not exclusive and (2) the
client is informed of the existence and
nature of the agreement. A lawyer who
receives such referrals may not pay for the
referral.

Any advertisement for a lawyer or law firm


must include the name and office address
of a lawyer or firm responsible for the
content.

7.3 Direct contact with prospective clients


Model Rule 7.3 sets out the ethical requirements for lawyers when communicating directly with
prospective clients. As the comment explains, the underlying concern is to avoid exercising undue
influence, intimidation, and over-reaching when obtaining new clients. Model Rule 7.3, cmt. [1].

(a) [1] A lawyer shall not by in-person, live

telephone or real-time electronic contact [2]


solicit professional employment [3] from a
prospective client [4] when a significant motive
for the lawyer's doing so is the lawyer's
pecuniary gain, [5] unless the person contacted:
(1) [6] is a lawyer; or
(2) [6] has a family, close personal, or prior

professional relationship with the lawyer.


(b) [1] A lawyer shall not solicit professional
employment [2] from a prospective client [3]
by written, recorded or electronic
communication or by in-person, telephone or
real-time electronic contact [4] even when not
otherwise prohibited by paragraph (a), if:
(1) [5] the prospective client has made known
to the lawyer a desire not to be solicited by
the lawyer; or
(2) [5] the solicitation involves coercion,
duress or harassment.
(c) [1] Every written, recorded or electronic
communication from a lawyer [2] soliciting
professional employment from a prospective
client [3] known to be in need of legal services
in a particular matter [4] shall include the
words Advertising Material on the outside
envelope, if any, and [5] at the beginning and
ending of any recorded or electronic
communication, [6] unless the recipient of the
communication is a person specified in
paragraphs (a)(1) or (a)(2).
(d) [1] Notwithstanding the prohibitions in
paragraph (a), [2] a lawyer may participate
with a prepaid or group legal service plan [3]
operated by an organization not owned or
directed by the lawyer that [4] uses in-person
or telephone contact to solicit memberships or
subscriptions for the plan [5] from persons who
are not known to need legal services in a
particular matter covered by the plan.

General prohibition against live solicitation:


whether in-person, by telephone, or by realtime electronic contact, such as chat rooms.
Class actions: the advertising and solicitation
riles do not prohibit communications
authorized by law, such as notice to members
of a class in class action litigation. See c.4.
Solicitation by third parties: may not use other
people to solicit for them. See 7.3, 8.4.
Prohibits any solicitation of someone who has
let the lawyer know he does not want to be
solicited, and solicitation that involves
coercion, duress, or harassment.

Required that targeted communications be


labeled advertising material when the
communication is with a person known to be
in need of legal services in a particular matter.

Authorizes lawyers to participate in prepaid


and group legal service plans generally, as long
as eh plan: (1) is not owned or directed by the
lawyers providing the services, and (2) does
not engage in solicitations targeted to persons
known to be in need of the particular legal
services covered by the plan. See c.6.

8.1 Bar admission and disciplinary matters


A lawyer has a general duty to maintain the integrity of the legal profession. In most states, a lawyer
MUST inform the appropriate professional authorities if he has knowledge that another lawyer has
acted in a manner that raises a substantial question as to the lawyers honesty, trustworthiness, or
fitness as a lawyer. A lawyer will be subject to discipline for not reporting when he is required to do
so. However, a mere rumor of wrongdoing by an attorney does not give rise to a duty for another
lawyer to report the rumor.
The rationale behind the suspension even for non-law related crimes is that committing a crime
demonstrates to the public that an attorney is unfit to be an officer of a court that is designed to
uphold the laws of a state.

[1] An applicant for admission to the bar, or a lawyer in connection with a bar admission application
or in connection with a disciplinary matter, shall not:
(a) [2] knowingly
Duty of candor and cooperation: imposes a duty of candor in connection
make a false
with all communications with admission to the bar as well as to lawyers;
statement of [3]
also the basis for disciplining a lawyer who ahs already been admitted by
material fact; or
the time his lack of candor during the application process is discovered.
See c.1.
Knowingly making a false statement: motive is irrelevant and should come
in only as a mitigating or aggravating factor
Materiality: a false statement does not violate 8.1(a) unless it concerns a
material fact.
Expunged convictions and sealed records: duty of disclosure in bar
applications can extend to information contained in sealed records.
(b) [2] fail to disclose a Knowing failure to respond: violated 8.1(b) only if it is knowing, which
fact necessary [3] to
may be inferred from the circumstances.
correct a
Fifth Amendment: a lawyers obligations under 8.1 are subject to the Fifth
misapprehension
Amendment, which protects any person from being compelled as a witness
known by the
against himself. The privilege may be invoked to avoid any compelled
person to have
disclosure that the speaker might reasonably apprehend could be used in a
arisen in the matter,
criminal prosecution.
or [3] knowingly
Lawyer must still respond: availability of the Fifth Amendment does not
fail to respond to a
relieve the lawyer of the duty to make some response to an inquiry from
lawful demand for
bar authorities.
information from
No protection against disciplinary sanctions: cannot be invoked as a
an admissions or
protection against disciplinary sanctions themselves.
disciplinary
authority, [4] except Adverse inference: a lawyers assertion of the Fifth Amendment privilege
in a disciplinary proceeding may result in an adverse inference being
that this Rule does
drawn in connection with the disciplinary charges. Claiming a Fifth
not require
Amendment privilege can be even more damaging in the admissions
disclosure of
context where the applicant has the burden to prove his good moral
information
character.
otherwise protected
by Rule 1.6.

8.3 (related rules: 1.6, 4.4)


Although Rule 8.3 does not require a lawyer to report his or her own misconduct, the lawyer must
report the misconduct of others even if doing so would implicate the lawyer's own conduct as well.
Rule 8.3 does not specify when the report must be made.
C2: while a lawyer does not have to disclose conduct protected by Rule 1.6, the lawyer should
encourage the client to consent to the disclosure where prosecution would not substantially
prejudice the clients interests.
C4: the duty to report does not apply to a lawyer retained to represent a lawyer whose
professional conduct is in question.

(a) [1] A lawyer who knows that [2]


If a lawyer knows that another lawyer or a judge has committed
another lawyer has [3] committed a
a violation of the Rules of Professional Conduct, and if the
violation of the Rules of
lawyer believes the violation raises a substantial question
Professional Conduct [4] that
about honesty or fitness, Rule 8.3 requires the lawyer to
raises a substantial question as to
report the misconduct. The duty to report misconduct under
that lawyer's honesty,
Rule 8.3 is nondiscretionary. It is therefore unethical merely
trustworthiness or fitness as a
to threaten to report.
lawyer in other respects, [5] shall Subsections (a) and (b) require reporting only when a lawyer
inform the appropriate professional
knows of another lawyer's or judge's misconduct.
authority.
Knows is defined as actual knowledge of the fact in
question, which may be inferred from circumstances.
Model Rule 1.0(f);
Model Rule 8.3 obliges lawyers to report only those violations
of the ethics rules that raise a substantial question about
honesty or fitness. Substantial is defined in the Model
Rules as of clear and weighty importance.
Subsections (a) and (b) do not specify what the appropriate
authority to receive a report is, but Comment [1] mentions
the need to initiate [a] disciplinary investigation, and
Comment [3] refers to making a report to the bar
disciplinary agency unless some other agency is more
appropriate in the circumstances.
(b) [1] A lawyer who knows that [2] a Requires that misconduct by a judge be reported to the
judge has [3] committed a
appropriate authority.
violation of applicable rules of
judicial conduct [4] that raises a
substantial question as to the
judge's fitness for office [5] shall
inform the appropriate authority.
(c) [1] This Rule does not require
Makes the duty to report misconduct subordinate to the
disclosure of information [2]
duty of confidentiality set forth in 1.6.
otherwise protected by Rule 1.6 or For reasons of social policy, Rule 8.3(c) exempts from the
information gained by a lawyer or
reporting requirement any information learned through
judge while participating in an
participation in an approved lawyers assistance program.
approved lawyers assistance
program.

8.4 (Misconduct)
8.4 is not limited to the lawyer-client relationship; it reaches conduct outside the practice of law.
Lawyers have been disciplined under Rule 8.4 for misconduct that occurred before bar admission.
A pattern of repeated offenses, even minor ones, can indicate indifference to a legal obligation.
A lawyer who is told to do something that the lawyer thinks is unethical has several options:
Accept the directions of the supervisor
Do more research to try to clarify the problem
Argue with the supervisor
Ask to be relieved form work on the matter
Discuss the problem with another superior
Resign (or be fired) from the employment

(a)

(b)

(c)

(d)

(e)
(f)

o
o [1] It is professional misconduct for a lawyer to:
[2] violate or
Violating or attempting to violate ethics rules: characterizes a violation of any of
attempt to violate
the ethics rules as an act of misconduct. Thus, in a disciplinary proceeding, a
the Rules of
finding that a lawyer violated any ethics rule may be accompanied by the
Professional
finding that the lawyer thereby violates 8.4(a). Additionally, it is misconduct
Conduct, [3]
even to attempt to violate an ethics rule.
knowingly assist or Assisting in violation, or violating through acts of another: prohibits a lawyer
induce another to
from knowingly assisting or inducing another to violate the ethics rules, or
do so, [4] or do so
from violating those rules through the acts of another.
through the acts of
another;
[2] commit a
Scope of rule: subjects a lawyer to discipline for engaging in certain types of
criminal act that [3]
criminal conduct, which reflects adversely on the lawyers honesty,
reflects adversely
trustworthiness, or fitness as a lawyer. See c.2.
on the lawyer's
Crimes committed in nonlawyer capacity: rule reaches criminal conduct by a
honesty,
lawyer whether or not the lawyer was acting as a lawyer at the time.
trustworthiness or Not all criminal conduct violates the rule: covers only criminal conduct that
fitness [4] as a
reflects adversely on the lawyers fitness to practice law.
lawyer in other
Conviction not necessary: lawyer need not be convicted of, or even charged
respects;
with, a crime to violate the rule.
o
Offenses covered: drug and alcohol offenses, crimes involving dishonesty or
o
fraud, sex offenses, violent crimes, tax law violations.
[2] engage in
Dishonesty toward clients: includes misappropriating client funds.
conduct [3]
Dishonesty toward tribunal: violates 3.3 and 8.4.
involving
Other crimes: plagiarizing, ghostwriting and undisclosed assistance to pro se
dishonesty, fraud,
litigants
deceit or
Deceiving others: including adverse parties, opposing counsel, members of law
misrepresentation;
firm, other nonclients, covert investigations, secretly recording conversations
o
of third parties, mining metadata
[2] engage in
o Includes: abusive or disruptive behavior, failure to comply with
conduct [3] that is
court rules and orders, failure to corporate with disciplinary
prejudicial to the
investigation, abuse of legal process and frivolous claims,
administration of
sexual misconduct, abuse of public office or private position of
justice;
trust, lying to tribunal, lack of competence or diligence
A lawyer who manifests bias or prejudice based on race, sex, religion, national
origin, disability, age, sexual orientation or socio-economic status is
engaging in conduct prejudicial to the administration of justice
[2] state or imply [3] an ability to influence improperly a [4] government agency or official or [5] to
achieve results by means that violate the Rules of Professional Conduct or other law; or
[2] knowingly assist a judge or judicial officer [3] in conduct that is a violation of applicable rules of
judicial conduct or other law.

S-ar putea să vă placă și